Location via proxy:   [ UP ]  
[Report a bug]   [Manage cookies]                
Download as pdf or txt
Download as pdf or txt
You are on page 1of 41

Sample test questions

Krok 2
Medicine
()
Терапевтичний профiль 2

1. A patient with Morgagni-Adams-Stokes resonance and absence of respiration.What is


syndrome has fainted when walking up the the most likely diagnosis in this case?
stairs. The skin is pale, the pupils are dilated,
tonoclonic spasms are observed, rib cage is A. Spontaneous pneumothorax
immobile. Make the diagnosis: B. Hemothorax
C. Lobar pneumonia
A. Clinical death D. Pulmonary embolism
B. Social death E. Acute pleurisy
C. Preagony
D. Agony 6. A 34-year-old man on the 3rd day
E. Biological death of ceftriaxone treatment for acute otitis
(daily dosage - 2 grams) developed diarrhea
2. A 32-year-old welder complains of weakness occurring 5-6 times per day. Feces are without
and fever. His illness initially presented as mucus or blood admixtures. Temperature is
tonsillitis one month earlier. On examination: 36.6o C . Gregersen reaction (occult blood in
temperature - 38.9o C , respirations - 24/min., feces) is negative. Stool culture detected no
pulse - 100/min., blood pressure - 100/70 mm pathogenic germs. What is the most likely
Hg, hemorrhages on the legs, enlargement of cause of diarrhea in this case?
the lymph nodes. Complete blood count shows
Hb- 70 g/L, RBC- 2.2 · 1012 /L, WBC- 3.0 · 109 /L A. Antibiotic-associated diarrhea
with 32% of blasts, 1% of eosinophiles, 3% of B. Intestinal dysbiosis
bands, 36% of segments, 20% of lymphocytes, C. Bacterial overgrowth syndrome
and 8% of monocytes, ESR- 47 mm/hour. D. Ulcerative colitis
What is the cause of anemia in this case? E. Crohn’s disease (regional enteritis)
A. Acute leukemia 7. A 24-year-old patient visited a doctor
B. Chronic lympholeukemia complaining of enlargement of his
C. Aplastic anema submaxillary lymph nodes. Objectively:
D. B12 -deficient anemia submaxillary, axillary and inguinal lymph
E. Chronic hemolytic anemia nodes are enlarged. Chest X-ray shows:
enlarged lymph nodes of mediastinum. Blood
3. After a 5-day-long celebration of his test: erythrocytes - 3.4 · 1012 /L, Hb- 100 g/L,
daughter’s wedding a 65-year-old patient blood colour index - 0.88, platelets - 190·109 /L,
”saw” in his yard many cats, chickens, and rats.
He tried to chase them away, but was scared leucocytes - 7.5·109 /L, eosinophiles - 8%, band
off when the animals started scolding him and neutrophiles - 2%, segmented neutrophiles -
tried to harm him. What is the likely diagnosis? 67%, lymphocytes - 23%, ESR - 22 mm/hour.
What test must be prescribed to verify the
A. Delirium tremens cause of lymphadenopathy?
B. Senile psychosis
C. Schizophrenia A. Open biopsy of the lymph nodes
D. Organic brain syndrome B. Abdominal US
E. Reactive hallucinosis C. Mediastinum tomography
D. Puncture biopsy of the lymph nodes
4. 5 weeks after hypothermia a 22-year- E. Sternal puncture
old patient developed fever, weakness,
muscle pain, inability to move independently. 8. A 37-year-old woman complains of
Objectively: tenderness, induration of headaches, nausea, vomiting, spasms. The
shoulder and shin muscles, restricted active onset of the disease occurred the day before
movements, erythema on the anterior surface due to her overexposure to cold. Objectively:
of the chest. There is a periorbital edema with fever up to 40o C ; somnolence; rigid neck;
heliotropic erythema. Gottron’s sign is present. Kernig’s symptom is positive on the both
What investigation is required for the diagnosis sides; general hyperesthesia. Blood test:
verification? leucocytosis, increased ESR. Cerebrospinal
fluid is turbid, yellow-tinted. What changes of
A. Muscle biopsy the cerebrospinal fluid are most likely?
B. Aminotransferase activity
C. Pneumoarthrography A. Neutrophilic pleocytosis
D. ASO titer B. Lymphocytic pleocytosis
E. Rheumatoid factor C. Blood in the cerebrospinal fluid
D. Xanthochromia in the cerebrospinal fluid
5. During physical exertion a man suddenly E. Albuminocytological dissociation
developed acute chest pain on the right and
dyspnea. Objectively he assumes forced half- 9. A 44-year-old patient complains of difficult
sitting position in the bed, presents with diffuse urination, sensation of incomplete urinary
cyanosis, resting tachypnea of 38/min., the bladder emptying. Sonographic examination
right side of the thorax is enlarged and does of the urinary bladder near the urethra
not participate in the respiratory process; entrance revealed an oval well-defined
percussion on the right reveals tympanic hyperechogenic formation 2x3 cm large
that was changing its position during the
Терапевтичний профiль 3

examination. What conclusion can be made?


A. Progressive angina pectoris
A. Stone B. First-time angina pectoris
B. Malignant tumour of the urinary bladder C. Stable NYHA functional class II angina
C. Urinary bladder polyp pectoris
D. Prostate adenoma D. Variant angina pectoris
E. Primary ureter tumour E. Acute cardiac infarction
10. A 25-year-old woman complains of fatigue, 14. During appointment with the doctor
dizziness, hemorrhagic rashes on the skin. She a man complains of painful itching rashes
has been presenting with these signs for a that appeared on his skin under the beard
month. Blood test: erythrocytes - 1.0 · 1012 /L, and moustache one year ago, with frequent
Hb- 37 g/L, colour index - 1.1, leukocytes - exacerbations occurring throughout the year.
1.2 · 109 /L, platelets - 42 · 109 /L. What analysis Objectively the skin of the facial hair growth
would be the most advisable for diagnosis- areas is bluish-purple, thickened, with pustules,
making in this case? erosions, and scabs covering its moist surface.
The fistulous tracts are surrounded by the
A. Sternal puncture (bone marrow biopsy) area of loose pink-red granulation and
B. Splenic biopsy discharge pus. The skin resembles mulberry
C. Liver biopsy in appearance. Make the diagnosis:
D. Coagulation studies
E. US of the gastrointestinal tract A. Sycosis
B. Acne rosacea
11. A 35-year-old man complains of rapidly C. Lupus erythematosus
incresing fatigue, palpitations, ”visual snow”, D. Deep trichophytosis
dizziness. He has a history of peptic ulcer of the E. Tuberculous lupus
stomach. Objectively the skin is pale. Vesicular
respiration is observed in the lungs. Systolic 15. A 60-year-old man presents with ischemic
murmur is detected over the cardiac apex, heart disease and heart failure of the IV
heart rate is 100/min., BP is 100/70 mm Hg. class according to NYHA (New York Heart
The epigastrium is slightly tender on palpation. Association) that manifests as dyspnea at
Blood test: erythrocytes - 3.2 · 1012 /L, Нb- 100 rest. There are moist crackles in the patient’s
g/L, color index - 0.94. What type of anemia is lungs. Liver +4 cm, lower limbs are swollen.
the most likely present in this case? Ejection fraction is 25%. What sign is the most
indicative when determining functional class
A. Posthemorrhagic anemia of heart failure according to NYHA?
B. Sideroblastic anemia
C. Iron-deficiency anemia A. Degree of dyspnea
D. Hemolytic anemia B. Moist crackles in the lungs
E. Hypoplastic anemia C. Swollen lower limbs
D. Decrease of ejection fraction
12. A 35-year-old patient has been suffering E. Extent of liver enlargement
from an illness for 3 days. 5 days ago he
returned from a trip to Africa. The onset of 16. A patient suffering from infiltrative
disease was accompanied by fever up to 40o C , pulmonary tuberculosis was prescribed
chills, acute headache, myalgia. In the axillary streptomycin, rifampicin, isoniazid, pyrazinamide,
region the lymph node enlarged up to 3x6 cm vitamin C. One month after the beginning of
can be palpated. The lymph node is dense, the treatment the patient started complaining
intensely painful, slightly mobile, without clear of reduced hearing and tinnitus. What drug has
margins; the skin over the node is hyperemic such a side effect?
and tight. Tachycardia is present. Make the A. Streptomycin
preliminary diagnosis: B. Isoniazid
A. Plague C. Rifampicin
B. Sepsis D. Pyrazinamide
C. Tularemia E. Vitamin C
D. Lymphadenitis 17. A 39-year-old man complains of morning
E. Anthrax headaches, appetite loss, nausea, morning
13. A 52-year-old patient, who has been vomiting, periodic nasal hemorrhages. The
suffering from angina pectoris, for 2 weeks patient had a case of acute glomerulonephritis
has increasingly frequent pain attacks in the at the age of 15. Examination revealed rise
area behind his sternum and his need for of arterial pressure up to 220/130 mm Hg,
nitroglycerine has increased. Objectively: the skin hemorrhages on his arms and legs,
condition is of moderate severity. The skin is pallor of skin and mucous membranes. What
pale. Heart sounds are weakened, rhythmic. biochemical parameter is the most important
Heart rate is 84/min. ECG shows no signs of for making diagnosis in this case?
focal myocardial injury. What is the most likely
diagnosis?
Терапевтичний профiль 4

A. Blood creatinine center. What disease is most likely in the given


B. Blood bilirubin case?
C. Blood sodium
D. Uric acid A. Pneumonia complicated by an abscess
E. Fibrinogen B. Infiltrative tuberculosis
C. Peripheral pulmonary cancer
18. The doctor has an appointment with a D. Cystic echinococcosis
patient, who 2 days ago developed severe E. Pulmonary cyst
chest pain on the left, general weakness, high
temperature, and headache. Objectively along 22. A 16-year-old adolescent living in a rural
the 4th and 5th intercostal nerves on the left area has been bitten in the shin by a stray dog.
the skin is hyperemic and there are tight The wound is superficial. Regular vaccination
clusters of small vesicles filled with clear serous against tetanus was received 3 months ago.
content. What is the most likely diagnosis? What treatment tactics would be the most
advisable in this case?
A. Herpes zoster
B. Herpes simplex A. Antirabies vaccination
C. Streptococcal impetigo B. Antirabies immunoglobulin
D. Pemphigus C. Tetanus toxoid adsorbed
E. Dermatitis herpetiformis (Duhring’s disease) D. Antitetanus serum
E. Antitetanus immunoglobulin
19. For a week a 42-year-old patient has
been suffering from fever attacks followed 23. A 48-year-old patient was found to have
by high temperature, which occur every 48 diffuse enlargement of the thyroid gland,
hours. Body temperature raises up to 40o C and exophthalmia, weight loss of 4 kg in 2 months,
decreases in 3-4 hours with excessive sweating. sweating. Objectively: HR- 105/min, BP-
The patient presents with loss of appetite 140/70 mm Hg. Defecation act is normal. What
and general fatigue. The skin is pale and kind of therapy is recommended in this case?
sallow. The liver and spleen are enlarged and
dense on palpation. What method of diagnosis A. Mercazolil
verification would be most efficient? B. Radioactive iodine
C. Propranolol
A. Microscopy of blood smear and thick blood D. Lugol’s solution
film E. Thyroxine
B. Complete blood count
C. Bacteriological analysis 24. A 26-year-old woman is suspected to
D. Immune-enzyme assay suffer from systemic lupus erythematosus with
E. Microscopy of hanging blood drop systemic lesions of skin, vessels, joints, serous
tunics, and heart tissue that developed after
20. A 28-year-old patient is a drug addict. He photosensitization. The following is detected
has been sick for a year, when noticed general in blood analysis: LE cells, antibodies to native
weakness, increased sweating, and weight loss. ds-DNA, isolated anti-centromere antibodies,
He often had cases of respiratory diseases. rheumatoid factor is 1:100, Wassermann
Within the last 2 days he demonstrates reaction is positive, circulating immune
intermittent fever with profuse night sweating, complex is 120 units. What immunological
increased general weakness, developed indicators are considered to be specific to this
diarrhea with mucus and blood admixtures. On disease?
examination: polylymphadenopathy, herpetic
rashes in the oral cavity; on abdominal A. ds-DNA antibodies
palpation: the liver and spleen are enlarged. B. Rheumatoid factor
What is the most likely diagnosis? C. Anti-centromere antibodies
D. Immunoglobulin A
A. HIV-infection E. Increased circulating immune complex
B. Herpetic stomatitis
C. Chronic lymphatic leukemia 25. A woman came to a doctor with complaints
D. Colon cancer of increased body temperature up to 37,8o C
E. Chronic sepsis and moderately sore throat for the last 3
days. Objectively: mandibular lymph nodes
21. A 40-year-old patient has acute onset are enlarged up to 3 cm. Palatine tonsils are
of disease caused by overexposure to cold. hypertrophied, with gray coating that spreads
Temperature has increased up to 39o C . to the uvula and anterior pillars of the fauces.
Foul-smelling sputum is expectorated during What is the most likely diagnosis?
coughig. Various moist crackles can be
auscultated above the 3rd segment on the A. Oropharyngeal diphtheria
right. Blood test: leukocytes - 15, 0 · 109 /l, stab B. Infectious mononucleosis
neutrophils - 12%, ESR- 52 mm/hour. On X- C. Pseudomembranous (Vincent’s) tonsillitis
ray: in the 3rd segment on the right there is a D. Agranulocytosis
focus of shadow 3 cm in diameter, low density, E. Oropharyngeal candidiasis
with fuzzy smooth margins and a clearing in its 26. A group of 5 had been resting in a forest,
Терапевтичний профiль 5

they were drinking alcohol and eating canned the knee and elbow joints is observed. Pulse
mushrooms and cured fish. The next day two of is 106/min., rhythmic. Blood pressure is 90/60
them were hospitalized with disturbed vision, mm Hg. Cardiac borders are unchanged, heart
swallowing and respiration; the third one sounds are weakened, at the cardiac apex there
presented with acute general weakness and is soft systolic murmur. What factor would
dry mouth. The remaining two were healthy. be the most indicative of the likely disease
A tick was detected on the skin of one of etiology?
the healthy group members. What is the most
likely diagnosis? A. Anti-streptolysin O
B. C-reactive protein
A. Botulism C. Creatine kinase
B. Tick-borne encephalitis D. Rheumatoid factor
C. Alcohol poisoning E. Seromucoid
D. Mushroom poisoning
E. Lyme borreliosis 31. A 45-year-old man complains of cough
fits and tickling in his nasopharynx. He had
27. A 23-year-old man complains of facial been staying for 10 days in the polluted area
edema, headache, dizziness, low urinary created by the Chornobyl nuclear power plant
output, urine discoloration (dark red). These accident. Rhinoscopy shows signs of severe
complaints arose after the patient had had a nasopharynx irritation. What radionuclide is
case of acute tonsillitis. On examination there the cause of this irritation?
are facial edema, the skin is pale, temperature
is 37.4o C ; heart rate is 86/min., blood pressure A. Radioactive iodine
is 170/110 mm Hg. Heart sounds are muffled, B. Radioactive cesium
the II heart sound is accentuated over the C. Radioactive strontium
aorta. What etiological factor is the most likely D. Radioactive plutonium
in this case? E. Radioactive cobalt

A. Beta-hemolytic streptococcus 32. A 20-year-old patient complains of severe


B. Staphylococcus aureus headache, double vision, weakness, fever,
C. Streptococcus viridans irritability. Objectively: body temperature is
D. Streptococcus pyogenes 38.1o C , the patient is reluctant to contact,
E. Staphylococcus saprophyticus sensitive to stimuli. There are ptosis of the left
eyelid, exotropia, anisocoria S>D, pronounced
28. A 65-year-old man was diagnosed with meningeal syndrome. On lumbar puncture the
B12 -deficient anemia and the treatment was cerebrospinal fluid flowed out under a pressure
prescribed. A week later control blood test was of 300 mm Hg, the fluid is clear, slightly
performed. What would be the early indicator opalescent. 24 hours later there appeared
of the therapy effectiveness? fibrin film. Protein - 1.4 g/L, lymphocytes -
600/3 per mm3 , sugar - 0.3 mmol/L. What is
A. Increased number of reticulocytes the provisional diagnosis?
B. Increased hemoglobin level
C. Megaloblastic hematopoiesis A. Tuberculous meningitis
D. Normoblastic hematopoiesis B. Meningococcal meningitis
E. Increased erythrocyte number C. Lymphocytic (Armstrong’s) meningitis
D. Syphilitic meningitis
29. A 35-year-old woman complains of heart E. Mumps meningitis
pain (”aching and drilling”) occurring mainly
in the morning in autumn and spring and 33. A 32-year-old woman complains of
irradiating to the neck, back and abdomen; increasing spastic pains in her lower abdomen
rapid heartbeat; low vitality. Occurrence of that occur after emotional stress. Bowel
this condition is not associated with physical movements are intermittent: 2-3 defecations
activity. In the evening, the patient’s condition after waking in the morning alternate with
improves. Study of somatic and neurological constipations that last for 1-2 days. Objectively
status and ECG reveal no pathology. What body mass is retained, palpation of the
pathology is most likely to have caused these sigmoid colon is moderately painful. Hb- 130
clinical presentations? g/L, leukocytes - 5.2 g/L, ESR- 9 mm/hour.
Rectoromanoscopy is painful due to spastic
A. Somatization depression condition of the intestine, intestinal mucosa
B. Resting stenocardia is without changes. Intestinal lumen contains
C. Pseudoneurotic schizophrenia large amounts of mucus. What is the most
D. Neurocirculatory asthenia likely diagnosis in this case?
E. Hypochondriacal depression
A. Irritable bowel syndrome
30. An 18-year-old young man complains of B. Crohn’s disease (regional enteritis)
pain in his knee and elbow joints and body C. Nonspecific ulcerative colitis
temperature up to 39.5o C . One week and a half D. Acute mesenteric ischemia
earlier developed sore throat. On examination E. Malabsorption syndrome
his body temperature is 38.5o C . Swelling of
Терапевтичний профiль 6

34. A 37-year-old worker during a fire ended fatigue. He has been presenting with these
up in the area of high CO concentration. He symptoms for 3 months. He has been smoking
was delivered to a hospital in unconscious since early adolescence. Objectively to is
state. Objectively: the skin of his face 37.4o C , respirations are 26/min., pulse is
and hands is crimson. Respiration rate is 82/min., rhythmic, blood pressure is 130/85 mm
20/min. ECG: alterations specific for hypoxic Hg. The right side of the thorax lags behind
myocardium. Hourly diuresis is 40 ml. Blood in the respiratory process, dull percussion
test: erythrocytes - 4.5 · 1012 /L, Нb- 136 sound and acute decrease of breathing
g/L, color index - 0.9, ESR- 3 mm/hour, activity are observed there. X-ray shows
carboxyhemoglobin - 5%. What criterion homogeneous shadow of the lung field on the
allows determining the severity of the patient’s right with mediastinum displacement towards
condition? the affected side. What is the most likely
diagnosis?
A. Carboxyhemoglobin concentration
B. Respiratory disorders A. Central lung cancer
C. ECG results B. Exudative pleuritis
D. Extent of trophic disorders C. Pleuropneumonia
E. Development of chronic renal failure D. Pulmonary tuberculosis
E. Multiple bronchiectasis
35. A patient with suspected pheochromocytoma
has normal blood pressure in the periods 39. A 42-year-old woman complains of severe
between the atacks and a tendency pulsing headache in the frontoparietal area,
towards tachycardia. Urine test revealed vertigo, palpitations. She has been suffering
no pathologies. It was decided to use from hypertension for 3 years. Significant
a provocative test with histamine. What increase of BP occurs 2-3 times per month
medication should be prepared to provide and lasts for 3-8 hours. The left ventricle is
emergency care in case of positive test result? enlarged, heart sounds are clear, heart rate
- 105/min., BP- 225/115 mm Hg. ECG: signs
A. Phentolamine of left ventricular hypertrophy. What drug
B. Pipolphen would be the most effective for termination
C. Nifedipine of cerebral crisis attack?
D. Mesatonum
E. Prednisolone A. Labetalol
B. Hydrochlorothiazide (Hypothiazide)
36. A 46-year-old woman complains of C. Captopril
severe pain attacks in the right lumbar D. Losartan
area, which irradiate to the lower abdomen, E. Clonidine (Clophelin)
and nausea. This kind of pain attacks has
never been detected in the patient before. 40. A 45-year-old patient complains of pain
Plain abdominal X-ray reveals no pathologic in the epigastric region, left subcostal area,
shadows. Ultrasound detects a hyperechogenic abdominal distension, diarrhea, loss of weight.
mass 1.5 cm in diameter, which reflects sound He has been suffering from this condition for
wave, in the enlarged right renal pelvis. What 5 years. Objectively: the tongue is moist with
diagnosis is the most likely? white coating near the root; deep palpation of
abdomen reveals slight pain in the epigastric
A. Renal calculus region and Мауо-Robson’s point. Liver is
B. Benign renal tumor painless and protrudes by 1 cm from the costal
C. Renal cyst arch. Spleen cannot be palpated. What disease
D. Renal tuberculosis can be primarily suspected?
E. Malignant renal tumor
A. Chronic pancreatitis
37. A 60-year-old woman complains of B. Atrophic gastritis
unbearable pain in her right subcostal area. C. Peptic stomach ulcer
She has a history of acute pancreatitis. D. Chronic cholecystitis
Temperature is 38.2o C . Objectively her sclera E. Chronic enteritis
are icteric. There are no signs of peritoneal
irritation. Ortner’s and Gubergrits’ symptoms 41. For 4 days a 35-year-old man has been
are positive. Urine diastase is 320 U/L. What is treated in the resuscitation unit for acute renal
the most likely diagnosis? failure caused by compartment syndrome.
The patient is disoriented. ECG shows high
A. Exacerbation of chronic pancreatitis T waves and right ventricular extrasystoles.
B. Acute cholangitis His central venous pressure is 159 mmH2 O;
C. Chronic cholecystitis for the last 3 hours auscultation has been
D. Acute cholecystitis detecting isolated moist crackles in the lungs.
E. Pancreatic cancer Respirations are 32/min. Blood test: residual
nitrogen - 62 mmol/L, K + - 7.1 mmol/L, Cl− -
38. A 64-year-old man complains of cough with
expectoration consisting of blood-streaked 78 mmol/L, N a+ - 120 mmol/L, Ht- 0.32 L/L,
mucus, dyspnea, low grade fever and general Hb- 100 g/L, blood creatinine - 0.9 mmol/L.
In this case the most advisable would be to
Терапевтичний профiль 7

perform: condition for 2 years. Objectively she presents


with pale skin, subicteric sclera, and bright-
A. Hemodialysis red fissured tongue. Lymph nodes are not
B. Plasmasorption enlarged. Pulse - 100/min. BP- 105/70 mm Hg.
C. Hemosorption Liver +3 cm, the spleen cannot be palpated.
D. Plasmafiltration Blood test: erythrocytes - 1.2 · 1012 /L, Нb- 56
E. Ultrafiltration g/L, color index - 1.4, macrocytes, leukocytes
42. For three years a 31-year-old woman - 2, 5 · 109 /L, eosinophils - 1%, juvenile - 1%,
has been complaining of pain and swelling metamyelocytes - 1%, band neutrophils - 8%,
of her radiocarpal and metacarpophalangeal segmented neutrophils - 47%, lymphocytes -
articulations, their reduced mobility in the 38%, monocytes - 4%, reticulocytes - 0.1%,
morning, which persisted up to 1,5 hours. platelets - 100 · 109 /L, ESR- 30 mm/hour,
Two weeks ago she developed pain, swelling, indirect bilirubin - 26 mmol/L. What changes
and reddening of her knee joints, her body can be expected in the bone marrow puncture
temperature increased up to 37,5o C . The material?
treatment was belated. Examination of
the internal organs revealed no pathologic A. Prevalence of megaloblasts
alterations. Diagnosis of rheumatoid arthritis B. Increased number of sideroblasts
was made. What alterations are the most likely C. Erythroid hyperplasia
to be visible on the arthrogram? D. Presence of blast cells
E. Prevalence of lymphoid tissue
A. Joint space narrowing, usuration
B. Joint space narrowing, subchondral 46. A 58-year-old woman complains of
osteosclerosis spontaneous bruises, weakness, bleeding gums,
C. Cysts in the subchondral bone dizziness. Objectively: the mucous membranes
D. Numerous marginal osteophytes and skin are pale with numerous hemorrhages
E. Epiphyseal osteolysis of various time of origin. Lymph nodes are
not enlarged. Ps- 100/min, BP- 110/70 mm Hg.
43. A 55-year-old woman, a cook, complains There are no alterations of internal organs.
of pain in her right knee joint that has been Blood test results: RBC- 3, 0 · 1012 /l, Нb- 92 g/l,
troubling her for a month and intensifies in color index - 0,9, anisocytosis, poikilocytosis,
the evening. Objectively she is overweight, the WBC- 10 · 109 /l, eosinophils - 2%, stab
knee joint is swollen, creaks during movement, neutrophils - 12%, segmented neutrophils -
palpation reveals localized pain. The 1st 68%, lymphocytes - 11%, monocytes - 7%,
metatarsophalangeal articulation is deformed ESR- 12 mm/h. What laboratory test should be
on the both feet. No changes in blood and performed additionally to make the diagnosis?
urine are detected. What should be visible on
the patient’s X-ray? A. Platelets
B. Reticulocytes
A. Joint space narrowing, marginal osteophytes C. Clotting time
B. Osteoporosis, joint space narrowing, singular D. Osmotic resistance of erythrocytes
usurations E. Fibrinogen
C. Joint space narrowing, multiple usurations,
subluxations 47. A 45-year-old woman complains of
D. Epiphyseal erosions, bony ankylosis paroxysmal intolerable facial pain on the left
E. Joint space narrowing, round bone defects with attacks that last for 1-2 minutes. Attacks
are provoked by chewing. The disease onset
44. A 52-year-old woman has been suffering was two month ago after overexposure to
for 2 years from dull, occasionally exacerbating cold. Objectively: pain at the exit points of
pain in her right subcostal area, occurring after the trigeminal nerve on the left. Touching near
eating high-fat foods, bitter taste in her mouth the wing of nose on the left induces new pain
in the morning, constipations, and flatulence. attack with tonic spasm of the facial muscles.
Objectively she has excess weight, her body What is the most likely diagnosis among those
temperature is 36.9o C ; there is a coating listed?
on the root of her tongue; the abdomen is
moderately distended and painful in the area A. Trigeminal neuralgia
of gallbladder projection. What examination B. Glossopharyngeal neuralgia
would be the most helpful for diagnosis- C. Temporomandibular joint arthritis
making? D. Facial migraine
E. Maxillary sinusitis
A. Ultrasound
B. Duodenal intubation 48. A 28-year-old man complains of skin
C. Cholecystography rash and itching on the both of his
D. Duodenoscopy hands. The condition persists for 1,5 years.
E. Liver scanning The exacerbation of his condition he
ascribes to the occupational contact with
45. A 57-year-old woman complains of formaldehyde resins. Objectively: lesion foci
weakness, dyspnea, loss of appetite, and are symmetrically localized on both hands.
liquid feces. She has been suffering from this
Терапевтичний профiль 8

Against the background of erythema with A. Renal amyloidosis


blurred margins there are papulae, vesicles, B. Chronic glomerulonephritis
erosions, crusts, and scales. What is the most C. Acute glomerulonephritis
likely pathology? D. Heart failure
E. Chronic pyelonephritis
A. Occupational eczema
B. Idiopathic eczema 52. A 23-year-old patient had taken 1 g of
C. Allergic dermatitis aspirin to treat acute respiratory infection.
D. Simple contact dermatitis After that he developed an asthmatic fit
E. Erythema multiforme with labored expiration that was arrested by
introduction of aminophylline. The patient’s
49. A 25-year-old patient was delivered to medical history is not burdened with allergies.
an infectious diseases unit on the 3rd day The patient has undergone two surgeries for
of illness with complaints of headache, pain nasal polyposis in the past. What diagnosis is
in lumbar spine and gastrocnemius muscles, most likely?
high fever, chill. Objectively: condition of
moderate severity. Scleras are icteric. Pharynx A. Aspirin-induced asthma
is hyperemic. Tongue is dry with dry brown B. Atopic bronchial asthma
coating. Abdomen is distended. Liver is C. Infectious allergic bronchial asthma
enlarged by 2 cm. Spleen is not enlarged. D. Exercise-induced asthma
Palpation of muscles, especially gastrocnemius E. Symptomatic bronchospasm
muscles, is painful. Urine is dark in colour.
Stool is normal in colour. The most likely 53. A 36-year-old man has been complaining
diagnosis is: of marked weakness, low appetite, and nausea
for the last year, year and a half. Within the
A. Leptospirosis last year he has lost 10 kg of body mass.
B. Viral hepatitis type A The skin is darkened, especially on his face,
C. Malaria neck, and arms. Skin folds and nipples are
D. Infectious mononucleosis hyperpigmented, there are pigment spots on
E. Yersiniosis the patient’s inner thighs. Pulse is 60/min.,
blood pressure is 80/50 mm Hg. What is the
50. A 28-year-old woman complains of provisional diagnosis?
skin hemorrhages after minor traumas and
spontaneous appearance of hemorrhages on A. Adrenal insufficiency
the front of her torso and extremities. On B. Diabetes mellitus
examination: the skin is variegated (old and C. Chronic gastritis
new hemorrhages), bleeding gums. Blood D. Hemochromatosis
platelets - 20 · 109 /L; in the bone marrow there E. Cholestatic hepatitis
is increased number of megakaryocytes and 54. A 45-year-old man developed constricting
no platelet production. Treatment with steroid retrosternal pain that occurs during walks
hormones was effective. What is the likely at the distance of 200 m. Objectively heart
diagnosis? rate is 80/min., BP is 160/90 mm Hg. During
A. Idiopathic thrombocytopenic purpura cardiopulmonary exercise test at 50 W there is
B. Hemophilia a depression of S-T segment by 3 mm below
the isoline in V3-V4. What is the provisional
C. Rendu-Osler-Weber disease (Hereditary diagnosis?
hemorrhagic telangiectasia)
D. Disseminated intravascular coagulation A. Exertional angina pectoris, functional class
E. Acute vascular purpura III
B. Exertional angina pectoris, functional class
51. A 46-year-old man notes swollen legs, IV
weakness, sensation of fullness and heaviness C. Exertional angina pectoris, functional class
in the right subcostal area; it is the first II
occurrence of these signs in the patient. The D. Somatoform autonomic dysfunction,
patient has 20-year-long history of rheumatoid hypertension type
arthritis. The liver and spleen are enlarged E. Alcoholic myocardiodystrophy
and dense. Blood creatinine - 0,23 mmol/l,
proteinemia - 68 g/l, cholesterol - 4,2 mmol/l, 55. For the last 15 years a 48-year-old
urine specific gravity - 1012, proteinuria - patient has been working at the factory
3,3 g/l, isolated wax-like cylinders, leached producing synthetic resins. Lately he has been
erythrocytes in the vision field, leukocytes - complaining of significant general fatigue,
5-6 in the vision field. What is the most likely headaches, frequent urination (predominantly
complication? during the day), red color of urine. What
complication of benzene nitrocompounds
poisoning can be suspected?
Терапевтичний профiль 9

A. Malignant tumor of the urinary bladder 165 cm. The patient’s skin is clear, with even
B. Chronic cystitis distribution of subcutaneous fat; the thyroid
C. Chronic prostatitis gland is not enlarged; there are no menstrual
D. Acute glomerulonephritis cycle disturbances. What obesity prevention
E. Chronic pyelonephritis methods would be the most advisable in this
case?
56. A 44-year-old patient with postinfarction
cardiosclerosis presents with frequent heart A. Dietary treatment, graduated exercise
rate disorders and lower extremity edema. B. Intensive training regimen
Objectively: Ps- 95/min., irregular, 10-12 C. Gastroplasty or gastrojejunal shunt
extrasystoles per minute. BP- 135/90 mm Hg. D. Inhibitors of gastrointestinal lipases
The 1st heart sound at the apex is weakened. E. Anorectic drugs
Pulmonary respiration is rough. The liver
is enlarged +2 cm. ECG: irregular sinus 60. A 57-year-old woman complains of
rhythm, heart rate - 95/min., frequent polytopic a sensation of esophageal compression,
ventricular extrasystoles. What antiarrhythmic palpitations, difficult breathing when eating
drug is advisable in this case for treatment and solid food, occasional vomiting with a full
prevention of extrasystole? mouth, ”wet pillow” sign at night for the
last 6 months. Objectively: body tempearture
A. Amiodarone - 39o C , height - 168 cm, weight - 72 kg,
B. Lidocaine Ps- 76/min, BP- 120/80 mm Hg. X-ray study
C. Mexiletine revealed considerable dilation of esophagus
D. Quinidine and its constriction in the cardial part. What
E. Novocainamide (Procainamide) pathology is the most likely to have caused
dysphagia in this patient?
57. A 60-year-old woman started feeling
weakness, vertigo, rapid fatigability during the A. Achalasia cardiae
last year. Recently she has developed dyspnea B. Primary esophageal spasm
and paresthesia observed. Objectively: skin C. Hiatal hernia
and mucous membranes are pale and icteric. D. Esophageal carcinoma
Lingual papillae are smoothed out. Liver and E. Reflux esophagitis
spleen are at the edge of costal arch. Blood
test: Hb- 70 g/L, erythrocytes - 1.7 · 1012 /L, 61. A 43-year-old woman complains of pain in
blood color index - 1.2, macrocytes. What drug the lumbar area, which irradiates to her left leg
can be prescribed on pathogenetic grounds? and aggravates on movement, and sensation of
numbness in this leg. Objectively palpation
A. Vitamin B12 of her shin and thigh is painful, there are
B. Vitamin B6 painful stretch symptoms of on the left and
C. Ascorbic acid gastrocnemius cramps. There is no sensory loss
D. Iron preparations or weakening of reflex responses. Make the
E. Vitamin B1 diagnosis:
58. After excessive consumption of fatty food A. Vertebrogenous lumbar ischialgia on the left
a 60-year-old woman suddenly developed B. Vertebrogenous radicular syndrome of L5-S1
pain in her right subcostal area, nausea, on the left
bile vomiting, sharp bitter taste in her C. Left-sided coxitis
mouth. In 2 days she developed jaundice, D. Endarteritis of the lower extremities
her urine darkened. Objectively: sclera and E. Spinal stroke
skin are icteric, the abdomen is distended, the
liver is enlarged by 3 cm, soft and painful 62. A patient with chronic pancreatitis
on palpation, Ortner’s, Murphy’s, Kehr’s, complains of diarrhea occurring up to 5 times
Zakharyin’s, Mayo-Robson’s signs are positive. per day (no blood traces), loss of body weight,
What diagnostic technique should be used in abdominal distention, dryness of skin, loss
the first place to confirm the diagnosis? of hair, thirst, bleeding gums, convulsions.
Complete blood count: leukocytes - 5.8 · 109 /L;
A. Ultrasound of the gallbladder and bile duct Hb- 86 g/L; ESR- 15 mm/g; Blood protein
B. Fibrogastroduodenoscopy test: total protein - 48 g/L; albumins - 28 g/L.
C. Abdominal X-ray What indicators of coprological analysis would
D. Radionuclide scanning of the liver and accompany this syndrom?
gallbladder
E. Laparoscopy A. Steatorrhea, creatorrhea
B. Large amount of mucus, amylorrhea
59. A 30-year-old woman complains of C. Large amount of starch grains and cellulose
increased body weight and problems with D. Gas bubbles, acid reaction
physical exertion. Her parents are of increased E. Large numbers of iodinophilous microbes
body weight as well; typical meals in their
family are high in fats and carbohydrates. 63. A 35-year-old patient complains of
Objectively her blood pressure is 135/80 mm heartburn, gasseous and sour eructation,
Hg, pulse is 89/min., weight is 87 kg, height is burning constricting pain behind the sternum
Терапевтичний профiль 10

and along the esophagus, developing when A. Neuroleptics


bowing the torso to the front. No previous B. Antidepressants
examination; the patient takes almagel at C. Tranquilizers
his own discretion, after which he notes D. Nootropics
improvement of his general state. Make the E. Mood stabilizers
provisional diagnosis:
68. A young woman suffering from seborrhea
A. Gastroesophageal reflux disease oleosa has numerous light-brown and white
B. Functional dyspepsia spots on the skin of her torso and shoulders.
C. Cardiospasm The spots have clear margins, branny
D. Ulcer disease of the stomach desquamation, no itching. What provisional
E. Ulcer disease of the duodenum diagnosis can be made?

64. A 34-year-old man is being treated for A. Pityriasis versicolor


schizophrenia exacerbation in a psychiatric B. Torso dermatophytosis
unit. Objectively: the patient remains in bed, C. Seborrheic dermatitis
is sluggishly mobile, unresponsive, does not D. Pityriasis rosea
react to questions. His position is unvaried, E. Vitiligo
hypomimic, snout reflex and Dupre’s symptom
are present, muscles exhibit waxy flexibility. 69. A man came to an urologist with complains
He has been remaining in this state for of painful urination, discharge from urethra.
approximately a week. Feeding is parenteral. The patient has been suffering from this
Determine the neuromotor disturbance: condition for a week. Objectively: hyperemic
urinary meatus, edema, purulent discharge.
A. Catatonic stupor Microscopy of smears detected gram-negative
B. Depressive stupor bacteria. Specify the diagnosis:
C. Psychogenic stupor
D. Anergic stupor A. Acute gonorrheal urethritis
E. Exogenic stupor B. Trichomonas urethritis
C. Candidal urethritis
65. After overexposure to cold a 45-year-old D. Chlamydial urethritis
woman developed acute pain in her suprapubic E. Chronic gonorrhea
and lumbar areas during urination, sharp pains
at the end of urination, false urges to urinate. 70. A patient is 28 years old. He has been
Urine is turbid with blood streaks. The doctor suffering from mental disorder since he was
suspects urinary tract infection. What results 22. His current condition has changed acutely:
of laboratory analysis would be the most for 3 days the patient has been refusing to
indicative of such infection? leave his home. He claims that there is a
”telepathy” occurring between him and other
A. Leukocyturia, gross hematuria people, through which he receives ”thoughts
B. Gross hematuria of strangers” and transmits his own thoughts
C. Increased blood creatinine and blood urea for everyone to hear. He thinks his thoughts
D. Daily proteinuria under 3.0 and actions are manipulated through this
E. Daily proteinuria over 3.0 ”telepathy”. Make the preliminary diagnosis:

66. A 35-year-old patient developed an A. Paranoid schizophrenia


epileptic attack with tonoclonic spasms that B. Depressive episode
lasted for 3 minutes. After the attack the C. Catatonic episode
patient fell asleep but in 5 minutes the second D. Organic delirium
attack occurred. The first step of emergency E. Acute reaction to stress
aid would be to:
71. A 35-year-old woman has gained 20 kg
A. Ensure patency of airways weight within a year with the normal diet.
B. Take blood from the vein for analysis She complains of chills, sleepiness, shortness
C. Introduce diazepam intravenously of breath. The patient’s mother and sister
D. Prescribe antiepileptic drugs are corpulent. Objectively: height - 160 cm,
E. Administer chloral hydrate via enema weight - 92 kg, BMI- 35,9. Obesity is uniform,
there are no striae. The face is amimic. The
67. A 23-year-old woman has been suffering skin is dry. The tongue is thickened. Heart
from a mental disease since the age of 18, the sounds are muffled. HR- 56/min, BP- 140/100
course of disease has no remission periods. mm Hg. The patient has been suffering from
At a hospital the patient mostly presents amenorrhea for 5 months, has constipations.
with non-purposeful foolish excitation: she TSH- 28 mcIU/l (norm is 0,32-5). Craniogram
makes stereotypic grimaces, exposes herself, shows no pathology. What is the etiology of
publicly masturbates with a loud laughter, obesity?
repeates stereotypical abusive shouts. The
patient should be prescribed:
Терапевтичний профiль 11

A. Hypothyroid Barrel chest is observed, on percussion there


B. Hypo-ovarian is vesiculotympanitic (bandbox) resonance,
C. Hypothalamic-pituitary sharply weakened vesicular respiration on
D. Alimentary and constitutive both sides, moist crepitant crackles in the
E. Hypercorticoid lower segments of the lungs. Heart sounds are
72. A patient complains of painless ”sores” weakened, the II heart sound is accentuated
on his penis and inguinal lymph nodes over the pulmonary artery. The liver is +3 cm.
What complicated the clinical course of COPD
enlargement. Synthomycin emulsion that the
patient have been applying to the ”sores” was in this patient?
ineffective. Objectively: on the inner leaf of A. Chronic pulmonary heart
the foreskin there are three closely situated B. Pulmonary embolism
rounded erosions, 0,5 cm in diameter, with C. Acute left ventricular failure
dense infiltration that can be palpated at their D. Diffuse pneumosclerosis
bases. Make the preliminary diagnosis: E. Community-acquired pneumonia
A. Primary syphilis 76. A 72-year-old man with pnaumonia
B. Herpes simplex (Herpes pro genitalis) complains of marked dyspnea, chest pain,
C. Candidiasis of the inner leaf of the foreskin severe cough with expectoration, to is 39.5-
D. Shingles 40o C , no urination for a whole day. Objectively
E. Erythema multiforme the patient is conscious. Respiratory rate is
36/min. Over the right lower pulmonary lobe
73. A 19-year-old student was urgently percussion sound is dull; on auscultation there
hospitalized due to marked dyspnea and chest is bronchial respiration and numerous moist
pain on the left. Her body temperature is crackles. Blood pressure is 80/60 mm Hg. Heart
38.8o C . She has been presenting with these rate is 120/min. Heart sounds are muffled,
signs for 3 days. Respiratory rate is 42/min., there is tachycardia. What tactics should the
shallow. Percussion sound is dull to the left family doctor choose in the management of
from the center of the scapula, no respiration this patient?
can be auscultated. The left heart border is
displaced outwards by 3 cm. Embryocardia A. Hospitalization into intensive care unit
and heart rate of 110/min are observed. B. Outpatient treatment
Palpation of the right subcostal area is painful. C. Treatment in the day patient facility
What urgent measures should be taken in this D. Hospitalization into pulmonology unit
case? E. Hospitalization into neurology unit
A. Urgent thoracocentesis 77. After the contact with chemicals a
B. Prescription of penicillin antibiotics plant worker has suddenly developed
C. Administration of furosemide stridor, voice hoarseness, barking cough,
D. Administration of cardiac glycosides progressing dyspnea. Objective examination
E. Referral into thoracic surgery unit reveals acrocyanosis. What is the provisional
diagnosis?
74. A 48-year-old man complains of fatigue,
excessive sweating, severe skin itching, A. Laryngeal edema
undulant fever, enlarged cervical and B. Laryngeal carcinoma
supraclavicular lymph nodes. Objectively: C. PATE
paleness of skin and mucosa, cervical lymph D. Pulmonary atelectasis
nodes are mobile, dense, elastic, walnut-sized, E. Pneumothorax
painless, not attached to the skin. Complete
blood count: erythrocytes - 3, 0 · 1012 /l, Hb- 78. A 58-year-old man complains of general
100 g/l, leukocytes - 14 · 109 /l, eosinophils - weakness, loss of 10 kg of weight within
6%, basophils - 3%, band neutrophils - 11%, 1,5 months, progressive pain in the lumbar
segmented neutrophils - 69%, lymphocytes - 7, region, increased blood pressure up to 220/160
mm Hg, low grade fever. Objectively: in the
monocytes - 4%, platelets - 280 · 109 /l, ESR- 37 right hypochondrium deep palpation reveals
mm/hour. What method should be applied to a formation with uneven surface and low
verify the diagnosis? mobility; veins of the spermatic cord and
A. Lymph node biopsy scrotum are dilated. Blood test results: Hb- 86
B. Sternal puncture g/l, ESR- 44 mm/h. Urine test results: specific
C. Muscle biopsy gravity - 1020, protein - 0,99 g/l, RBC cover the
D. Chest X-ray whole field of vision, WBC- 4-6 in the field of
E. Lumbar puncture vision. What is the provisional diagnosis?

75. A 72-year-old man complains of lower A. Renal tumour


extremity edema, sensation of heaviness in B. Urolithiasis
the right subcostal area, dyspnea in rest. For C. Acute pyelonephritis
over 25 years he has been suffering from D. Acute glomerulonephritis
COPD. Objectively: orthopnea, jugular venous E. Nephroptosis
distention, diffuse cyanosis, acrocyanosis. 79. Anamnesis of a 30-year-old patient
Терапевтичний профiль 12

includes closed thoracic injury. Lately the dyspnea and cardiac pain. He ascribes his
patient has been suffering from increasing disease to the case of influenza that he had 2
dyspnea, sensation of heaviness in the right weeks ago. Objectively he leans forward when
subcostal area, and heart rate disturbances. sitting. The face is swollen, cyanotic, cervical
Objectively: acrocyanosis, bulging cervical veins are swollen. Heart borders are extended
veins, ascites, edema of the lower extremities. on the both sides, heart sounds are muffled,
Heart auscultation reveals muffled heart heart rate = Ps = 118/min., BP is 90/60 mm Hg.
sounds, additional III heart sound is Blood test: ESR is 16 mm/hour. ECG shows
detected. Provisional diagnosis of constrictive low voltage. X-ray shows trapezoidal cardiac
pericarditis was made. What diagnostic silhouette and signs of pulmonary congestion.
technique would NOT confirm the diagnosis? Choose the treatment tactics:
A. US of abdomen A. Pericardial puncture (pericardiocenthesis)
B. Computer tomography B. Diuretics
C. Echocardiography C. Antibiotics
D. Magnetic resonance imaging D. Pericardectomy
E. Chest X-ray E. Glucocorticosteroids
80. A 72-year-old woman suffers from diabetes 84. A 53-year-old woman complains of weight
mellitus type II, concomitant diseases are loss up to 10 kg within the last 2 years,
stage II hypertension and stage IIB heart liquid foul-smelling stool two times a day that
failure. She takes metformin. Hypertensic poorly washes off the toilet, periodic bouts
crisis had occurred the day before, after which of nausea, girdle pain in the upper abdomen.
the patient developed extreme weakness, Objectively: pain in Gubergrits zone (on the
myalgias, thirst, dry mouth, polyuria. BP is right from navel) and at Mayo-Robson’s point.
140/95 mm Hg, heart rate is 98/min., no edemas Biochemical blood analysis: glucose - 3,2
or smell of acetone detected. What measures mmol/l, bilirubin - 16,5 mcmol/l, crude protein
should be taken to prevent development of - 56,4 g/l. Urine diastase/amylase - 426 g/h/l.
comatose state in the patient? D-xylose test (oral administration of 25 g of
A. Stop metformin, prescribe short-acting d-xylose) after 5 hours reveals 3 g of xylose in
insulin urine. The most likely diagnosis is:
B. Double the dosage of metformin A. Pancreatitis. Malabsorption syndrome
C. Apply hypotonic solution of sodium chloride B. Pseudomembranous colitis
D. Additionally prescribe long-acting insulin C. Nonspecific ulcerative colitis
E. Prescribe glibenclamide D. Irritable bowel syndrome
81. Survey radiograph of a 52-year-old worker E. Chronic gastritis
of an agglomeration plant (28-year-long record 85. A 15-year-old teenager has undergone
of service, the concentration of metal dust medical examination in military recruitment
is 22-37 mg/m3 ) shows mildly pronounced center. The following was revealed: interval
interstitial fibrosis with diffused contrasting systolic murmur at the cardiac apex, accent of
well-defined small nodular shadows. The the II heart sound over the pulmonary artery,
patient has no complaints. Pulmonary function tachycardia. What additional examination
is not compromised. What is the provisional method will be the most informative for
diagnosis? determining diagnosis?
A. Siderosis A. Echocardiography
B. Silicosis B. Electrocardiography
C. Anthraco-silicatosis C. X-ray
D. Silicatosis D. Phonocardiography
E. Anthracosis E. Rheography
82. A 59-year-old patient suffering from 86. A 25-year-old patient is not married and
hypertension was delivered to the hospital has sexual relations with several partners.
with complaints of acute headache, nausea, During the last 3 months he noticed small
recurrent vomiting. On examination she amount of mucoserous secretions produced
presents with acute meningeal symptom from urethra. Subjectively: periodical itching
complex. BP is 185/105 mm Hg. What or burning pain in urethra. Two months ago
additional examination would you recommend pain in knee join developed. Possibility of
to the patient in the first place? trauma or exposure to cold is denied by the
patient. During the last week eye discomfort
A. Lumbar puncture is noted - lacrimation and itching. What
B. Ventriculopuncture provisional diagnosis can be suggested?
C. Echoencephalography
D. Rheoencephalography
E. Electroencephalography
83. A 36-year-old man complains of marked
Терапевтичний профiль 13

A. Reactive arthritis color index - 0,92, leukocytes - 7, 4 · 109 /l,


B. Rheumatoid arthritis platelets - 240 · 109 /l, ESR- 11 mm/hour.
C. Seasonal pollinosis What measure would most effectively decrease
D. Bacterial nonspecific urethral conjunctivitis hemorrhaging in this case?
E. URTI with conjunctiva and joints affected
A. Cryoprecipitate
87. A 19-year-old woman complains of severe B. Aminocapronic acid
pain in the axillary crease. Condition onset C. Native plasma
occurred a week ago after her swimming in a D. Direct transfusion of donor blood
cold river and epilation. The next day a painful E. Platelet concentrate transfusion
”boil” appeared. The ”boil” was increasing
in size every day and became a plum-sized 91. A 28-year-old woman complains of nausea,
tumor. Upon examination there are nodular stomachache, pain in her tongue, and liquid
conical growths joined together detected, the feces. Three days ago she ate poorly salted
skin covering them is bluish-red in color. Some pike caviar. Objectively her skin is pale, the
nodules have fistulous openings producing tongue looks ”lacquered” (bald tongue). Pulse
thick purulent mass. Body temperature is is 100/min., with muffled heart sounds and
38, 5o C , general malaise. What is the most systolic murmur over the cardiac apex. Blood
likely diagnosis? pressure is 95/50 mm Hg. The liver is enlarged
by 3 cm. Hemogram shows anemia, eosinophils
A. Hydradenitis - 18%. Oval helminth eggs were detected in
B. Carbuncle feces. Make the provisional diagnosis:
C. Cutaneous tuberculosis
D. Necrotizing ulcerative trichophytosis A. Diphyllobothriasis
E. Pyoderma chancriformis B. Trichinosis
C. Teniasis
88. A woman complains of weight gain, chills, D. Taeniarhynchosis
edema, xeroderma, somnolence, difficulties E. Ascaridiasis
with focusing. Objectively: height is 165
cm; weight is 90 kg; body proportions are 92. A 51-year-old woman complains
of female type, to - 35,8o C , heart rate - of headache, trembling, paresthesiae,
58/min., BP- 105/60 mm Hg. Heart sounds palpitations, increased blood pressure up
are weakened, bradycardia is observed. Other to 280/160 mm Hg. The day before she
internal organs have no alterations. Thyroid experienced exhausting headache, vascular
gland cannot be palpated. Milk secretion pulsation, palpitations, asphyxia, stomachache,
from mammary glands is observed. Hormone unbearable fear of coming death. The patient
test revealed increased levels of thyroid- paled and broke out in cold sweat. In urine
stimulating hormone (TSH) and prolactin, and there is increased content of vanillylmandelic
decreased level of thyroxine ( 4 ). What is the acid. What disease causes such clinical
cause of obesity? presentation in the patient?
A. Primary hypothyroidism A. Pheochromocytoma
B. Secondary hypothyroidism B. Conn’s syndrome (primary
C. Prolactinoma hyperaldosteronism)
D. Hypopituitarism C. Cushing’s syndrome
E. Adiposogenital dystrophy D. Primary hypertension
E. Cushing’s disease
89. During hemotransfusion the patient
developed nausea, tremor, lumbar and 93. A 26-year-old patient with affective bipolar
retrosternal pain. On examination the skin disorder has developed a condition manifested
is hyperemic, later developed pallor; the by mood improvement, behavioural and sexual
patient presents with hyperhidrosis, labored hyperactivity, verbosity, active body language,
respiration, pulse is 110/min., BP is 70/40 mm reduced need for sleep. Which of the following
Hg. Urine is black colored. What complication drugs would be most effective in this case?
developed in the patient?
A. Neuroleptics with sedative effect
A. Posttransfusion shock B. Antidepressants with activating effect
B. Acute renal failure C. Neuroleptics with activating effect
C. Pulmonary embolism D. Tranquilizers
D. Anaphylactic shock E. Antidepressants with sedative effect
E. Hypotonic crisis
94. A 19-year-old patient complains of dyspnea
90. A 22-year-old man suddenly developed on exertion. He often has bronchitis and
extreme weakness, nausea, vomiting with pneumonia. Since childhood the patient
traces of blood. The patient is known to presents with cardiac murmur. Auscultation
suffer from peptiv ulcer disease of duodenum revealed splitting of the II sound above the
and hemophilia A. Objectively: heart rate - pulmonary artery, systolic murmur in the 3rd
102/min., BP- 100/60 mm Hg. Complete blood intercostal space at the left sternal border.
count: erythrocytes - 3, 2 · 1012 /l, Hb- 98 g/l, ECG detected right bundle branch block.
Терапевтичний профiль 14

What is the provisional diagnosis? diagnosis?


A. Atrial septal defect A. Nephrotuberculosis
B. Open ductus arteriosus B. Right renal cyst
C. Aortarctia C. Right renal carcinoma
D. Aortic stenosis D. Acute glomerulonephritis
E. Mitral insufficiency E. Chronic pyelonephritis
95. A 37-year-old man suffers from attacks 99. A 36-year-old man developed a disease
of unconsciousness, dyspnea during physical with acute onset 6 hours ago. The patient
exertion, periodical sensations of heart rate presents with pain in the epigastric, ileocecal,
disorder. Father of the patient died suddenly and paraumbilical areas, vomiting, weakness,
at the age of 45. Objectively: heart rate nausea, and body temperature of 38,5o C . Stool
is 90/min., BP is 140/90 mm Hg. On heart is liquid, profuse, frequent, retains fecal nature,
US: ejection fraction - 55%, significant foul-smelling, frothy, colored dark green. The
myocardium thickening of the left ventricle stomach is moderately distended and painful
and interventricular septum. What drug should on palpation. The patient attributes his disease
be prescribed for the treatment? to eating raw chicken eggs one day before the
clinical signs of the disease appeared. What is
A. Bisoprolol the most likely diagnosis?
B. Enalapril
C. Phenyhydinum (Nifedipine) A. Salmonellosis
D. Hydrochlorothiazide B. Shigellosis
E. Furosemide C. Typhoid fever
D. Cholera
96. A 60-year-old man complains of discomfort E. Food toxicoinfection
when swallowing solid food, which he has
been observing for a month. He changed his 100. After the celebratory feast that took
diet to semiliquid food products. At first the place the day before, a 35-year-old man was
discomfort had abated but later it renewed hospitalized with complaints of marked pain
despite the change in the diet. The patient within the I metatarsophalangeal articulation
developed gaseous eructation and hoarse on the right, which developed late in the
voice. What examination should be performed night, and impaired walking. Objectively: the
to clarify the diagnosis? metatarsophalangeal articulation is swollen,
hyperemic, hot to touch, painful on movement.
A. Esophagoscopy with biopsy In blood: erythrocytes - 5, 1 · 1012 /l, Нb- 155 g/l,
B. Urea breath test for H. pylori leukocytes- 13, 0 · 109 /l, ESR- 50 mm/hour,
C. Diurnal variations of ECG parameters CRP- 46 mg/dl, uric acid - 720 mcmol/l. X-ray
D. Esophageal pH monitoring of feet articulations: osteoporosis, narrowing
E. Abdominal US of interarticular spaces, numerous punched-
97. After significant physical exertion a 66- out erosions. Make the preliminary diagnosis:
year-old man with deep vein thrombosis A. Gout
of the extremities developed shortness of B. Osteoarthritis
breath, intense pain in the chest on the left, C. Reactive arthritis
marked palpitations. The patient’s condition D. Rheumatoid arthritis
is grave, his face is cyanotic, the cervical E. Psoriatic arthritis
veins are swollen, BP is 60/40 mm Hg. What
investigation method would be the most 101. A patient has gradually lost consciousness.
advisable in this case? The skin is pale and dry. There is smell of
ammonia from the mouth. Respirations are
A. Selective angiopneumography deep and noisy. Heart sounds are muffled,
B. Chest X-ray pericardial friction rub is present. Blood
C. Echocardiography pressure is 180/130 mm Hg. Blood test: Нb-
D. Magnetic resonance imaging of the chest
E. Fiber-optic bronchoscopy 80 g/L, leukocytes - 12 · 109 /L, blood glucose
- 6.4 mmol/L, urea - 50 mmol/L, creatinine
98. A 32-year-old woman complains of - 1200 mcmol/L, blood osmolality - 350
general fatigue, low-grade fever persisting mOsmol/kg H2 O. No urinary excretion. Make
for 4 months, lumbar pain, and dysuria. the diagnosis:
Anamnesis includes frequent acute respiratory
diseases, overexposure to cold, low-calorie A. Uremic coma
diet, a case of pulmonary tuberculosis in B. Hyperglycemic coma
childhood. Clinical urine analysis: pH- 4.8, C. Acute renal failure
leukocyturia, hematuria. Complete blood D. Acute disturbance of cerebral circulation
count: leukocytosis, lymphocytosis, increased E. Hyperosmolar coma
ESR. Urography concludes: dilatation of renal 102. A 26-year-old woman has been
pelvis and calyceal system of both kidneys, undergoing treatment for community-acquired
foci of calcification in the projection of right pneumonia for 10 days. It is known that her
kidney parenchyma. What is the most likely
Терапевтичний профiль 15

husband had been treated for drug addiction. 106. A 38-year-old woman has been working
Sequential intravenous administration of as a milker for 15 years. She made
Amoksiklav (Amoxicillin+Clavunate) + an appointment with the doctor due to
Levofloxacin combination and vancomycin in development of red rashes on her hands,
the prescribed dosage was ineffective. Within predominantly in the interdigital space. The
the last two days the patient’s dyspnea and rashes are weeping, itching, and expanding
intoxication acutely exacerbated, bilateral on her skin. Examination of her hands shows
pulmonary infiltrates are observed. What her nail plates to be yellow and brittle. These
is the most likely cause of the medication presentations aggravate during work. Make
ineffectiveness? the provisional diagnosis:
A. HIV infection and pneumocystic pneumonia A. Occupational eczema
B. Tuberculosis mycobacterium infection with B. Scabies
development of tuberculosis C. Pemphigus
C. Idiopathic fibrosing alveolitis D. Pyoderma
D. Infection with polyresistant bacterial strains E. Dermatophytosis
E. Cancer metastases in the pulmonary tissues
107. An 18-year-old patient complains of skin
103. A 64-year-old man suddenly sensed rash. The patient has been suffering from
pain in his occipital area, dizziness, general this condition for 5 years. The first instance
weakness. He has a 15-year-long history of of this disease occurred after a car accident.
hypertension. Objectively the skin and mucosa Objectively: the patient presents with papular
are of normal color. Auscultation reveals rash covered in silvery scales, ”thimble”
vesicular respiration across the lung surface. symptom (small pits on the nails), affected
At the cardiac apex the I heart sound is joints. What is the most likely diagnosis?
weakened, the II heart sound is accentuated
over the aorta. Pulse is 84/min., blood pressure A. Psoriasis
is 180/100 mm Hg. Other body organs and B. Panaritium
systems are unaffected. What drug should be C. Onychomycosis
prescribed in the first place? D. Lupus erythematosus
E. Rheumatism
A. Captopril
B. Ramipril 108. A woman complains of frequent, liquid
C. Urapidil stool (up to 9-10 times per day) with mucus
D. Perindopril and blood admixtures, dull pain in the
E. Amlodipine hypogastrium, weight loss of 4 kg within
the last year. Objectively: malnutrition,
104. During winter epidemics of influenza dry skin, low turgor, aphthous stomatitis.
caused predominantly by virus А/California/04/2009 The stomach is soft, the sigmoid colon is
(H1N1), on the 2nd day after the disease onset spastic and painful on palpation. Occult
a 30-year-old hospitalized man presented with blood test is positive. Fibrocolonoscopy:
high fever, dry cough, myalgia, headache, and edema, hyperemia, mucosal granulation,
general weakness. What should be prescribed pseudopolyps, small ulcers with irregular
as etiotropic treatment in this case? edges. Make the diagnosis:
A. Neuraminidase inhibitors (Oseltamivir) A. Nonspecific ulcerative colitis
B. Antibiotics B. Chronic enterocolitis
C. Immunoglobulin C. Colon cancer
D. Interferon inducers D. Irritable bowel syndrome
E. Acyclovir E. Crohn’s disease (regional enteritis)
105. A 57-year-old patient complains of 109. A 48-year-old woman has been
dyspnea at rest. The patient presents with hospitalized due to development of
orthopnea, acrocyanosis, bulging cervical tachysystolic atrial fibrillation. She has lost
veins. On percussion: dull sound over the lower 5 kg of body weight within 2 months. On
lung segments; on auscultation: no respiratory palpation there is a node in the left lobe of
murmurs. Heart rate is 92/min. Right-sided the thyroid gland. What pathology resulted in
cardiac dilatation is observed. The liver is the development of this condition?
enlarged by 7 cm. Shins are swollen. Pleural
effusion is suspected. What indicator would A. Toxic nodular goiter
confirm the presence of transudate in this case? B. Aterosclerotic cardiosclerosis
C. Chronic thyroiditis
A. Total protein content in the pleural fluid D. Nontoxic nodular goiter
below 25 g/l E. Autoimmune thyroiditis
B. Presence of atypical cells
C. Total protein content in the pleural fluid 110. A 48-year-old woman developed
above 30 g/l insomnia, depressive mood, anxiety, fears
D. Specific gravity exceeding 1015 and suicidal thoughts after the death of her
E. Positive Rivalta’s test husband that occurred one month ago. During
Терапевтичний профiль 16

her stay in the hospital she speaks in a low A. Neuropathy of the facial nerve
voice, is depressed, anxious, avoids sleeping, B. Neuropathy of the trigeminal nerve
refuses to eat. What medications should be C. Trigeminal ganglionitis
prescribed in this case? D. Neuropathy of the oculomotor nerve
E. Ischemic stroke
A. Antidepressants
B. Antipsychotics 115. A 37-year-old man working as a typesetter
C. Group B vitamins in a print shop complains of rapid fatigability,
D. Nootropics paroxysmal attacks of stomachache, weak
E. Anticonvulsants drooping hands. Examination of neurological
status revealed hypotrophy of the forearm
111. A 32-year-old woman complains of muscles. Carporadial reflexes are sharply
episodes of intense fear that occur without weakened. Sensitivity is not disturbed.
visible cause and last for 10-20 minutes; the Gums present with dark blue border. What
episodes are characterized by rapid pulse, neurological pathology is it?
sweating, labored breathing, and vertigo.
Specify the likely diagnosis: A. Lead polyneuropathy
B. Guillain-Barre syndrome (postinfectious
A. Panic disorder polyneuritis)
B. Paranoid syndrome C. Shingles
C. Manic syndrome D. Ulnar neuropathy
D. Simple schizophrenia E. Brachial plexitis
E. Claustrophobia
116. A 44-year-old woman has undergone
112. A 39-year-old man suffers from chronic subtotal thyroid resection due to diffuse
adrenal insufficiency and receives replacement toxic goiter. On the second day after the
glucocorticoid therapy (hydrocortisone - 15 surgery the patient’s condition deteriorated;
mg/day). He is to undergo elective surgery she developed palpitations, dyspnea, sweating,
for calculous cholecystitis. What medication and diarrhea, and became fearful The patient
adjustment should be made on the day of the is anxious, her skin is moist and hot to the
surgery to prevent the development of acute touch. Her temperature is 39.2o C . Heart
adrenal insufficiency? sounds are muffled, tachycardia is observed,
pulse is 160/min., blood pressure is 85/40 mm
A. Increase the dosage by 2-3 times Hg. The stomach is soft and painless. What
B. Cancel the drug for the day of the surgery should be measured to clarify the patient’s
C. Add mineralocorticoid condition?
D. Add antibiotic
E. Prescribe large volume intravenous fluid A. Thyroid hormones
infusion B. Blood sugar
C. 17-KS and 17-OCS in urine
113. A resuscitation unit received a 46-year-old D. Blood epinephrine and norepineprine
woman, who has been suffering from diabetes E. Serum transaminases
mellitus type 1 for approximately 30 years.
Objectively: the skin is pale, heart sounds 117. The dermatologist has an appointment
are weakened, BP is 170/100 mm Hg, lower with a 30-year-old man that complains
limbs are markedly swollen. Blood creatinine of severely itching rashes that especially
- 1125 mcmol/l, urea - 49,6 mmol/l, potassium disturb him at night. The rashes developed
- 6.3 mmol/l, glucose - 7,6 mmol/l, glomerular 2 weeks ago, after he had returned from
filtration rate - 5 ml/min. What treatment is a travel. Objectively on the lateral surfaces
indicated for the patient in the first place? of his fingers, hands, wrists, elbows, lower
abdomen, genitals, and thighs there are paired
A. Hemodialysis papulovesicles, single pustules, and scratch
B. Kidney transplantation marks. What disease can be suspected?
C. Hemofiltration
D. Enterosorption A. Scabies
E. Conservative detoxification therapy B. Pyoderma
C. Dermatitis
114. After a long drive with the window open D. Eczema
a man developed facial asymmetry; he cannot E. Shingles
close his right eye, his right nasolabial fold is
smoothed out, movements of expression are 118. A 28-year-old man, a teacher, after
absent on the right, there is a disturbance emotional stress developed painful muscle
of gustatory sensation in the tongue on the spasms in his right hand that occur during
right. No other neurological pathologies were writing; now he has to hold the pen between
detected. What disease can be provisionally the second and third fingers. He has no
diagnosed in this patient? problems with typing or writing on the
blackboard; no other motor disturbances or
neurological pathologies are detected. What is
the most likely diagnosis?
Терапевтичний профiль 17

122. A 60-year-old man presents with


A. Writer’s cramp subcompensated viral liver cirrhosis (HCV),
B. Cortical agraphia Child-Pugh class B. What tactics should be
C. Parkinsonism chosen regarding the vaccination against
D. Neuropathy of the right ulnar nerve influenza in this case?
E. Neuropathy of the right radial nerve
A. Scheduled yearly vaccination
119. A 43-year-old man, who has been B. In case of influenza outbreak
abusing alcohol and suffering from pulmonary C. Combined with antiviral drugs
tuberculosis, in the course of two weeks D. Contraindicated due to disease progression
gradually developed general weakness, stage, as shown by Child-Pugh class
headache, diplopia, vomiting. Objectively: E. Contraindicated due to elderly age of the
ptosis on the left, anisocoria S>D, exotropia patient
of the left eye, neck stiffness; Kernig’s
and Brudzinski’s signs are positive. In 123. A 26-year-old man complains of chills,
cerebrospinal fluid: lymphocytic pleocytosis, rhinitis, dry cough, and fever up to 38o C .
low glucose, precipitation of cerebrospinal Examination shows him to be in a moderately
fluid resulted in production of fibrin film. What severe condition; there are small pale pink
is the most likely diagnosis? non-merging spots on the skin of his back,
abdomen, and extremities. Palpation reveals
A. Tuberculous meningitis enlarged occipital and axillary lymph nodes.
B. Subarachnoid hemorrhage No information about vaccination history
C. Brainstem encephalitis could be obtained. What is the likely etiology
D. Acute myelitis of this disease?
E. Basal arachnoiditis
A. Rubella virus
120. A 45-year-old woman has been suffering B. Epstein-Barr virus
from rheumatoid arthritis for 10 years and C. Streptococcus
takes methotrexate twice a week. What D. Mumps virus
statement regarding vaccination against E. Neisseria meningitis
pneumococci (23-valent vaccine) would
conform to the recommendations for the 124. A 40-year-old man developed fever up
management of rheumatoid arthritis issued by to 37.5o C and macular rash 10 days after the
the European League Against Rheumatism in first dose of MMR (Measles-Mumps-Rubella)
2010? vaccine was administered. The vaccination was
considered necessary as there was a measles
A. Vaccination is recommended outbreak in the city and the patient had not
B. Vaccination is not recommended received MMR vaccination in his childhood. Is
C. Vaccination is contraindicated to the patients revaccination with MMR vaccine possible?
who take methotrexate
D. Vaccination is contraindicated in cases when A. Possible
inflammatory process is active B. Forbidden
E. Vaccination necessitates increase in the C. After a course of glucocorticoids treatment
dosage of the long-term medicines D. Simultaneously with antihistamines
E. Under supervision in the infectious diseases
121. A 65-year-old woman was diagnosed with inpatient ward
the following: chronic rheumatic heart disease,
I degree of rheumatic activity; combined mitral 125. A 26-year-old man is undergoing a
heart disease with prevalence of III degree regular check-up. One year ago he had
stenosis; heart failure IIA with retained left a case of tonsillar diphtheria complicated
ventricular ejection fraction, functional class with myocarditis. Presently his condition is
III (NYHA). What tactics of vaccination satisfactory, no signs of cardiovascular failure;
against respiratory infections should be ECG shows first-degree atrioventricular block.
chosen to provide secondary prevention of What vaccine was administered to this man
exacerbations and to avoid heart failure according to his age?
decompensation in this patient?
A. Adsorbed diphtheria tetanus vaccine
A. Scheduled yearly vaccination against (modified)
influenza and pneumococci B. Acellular DPT vaccine
B. Vaccination should be combined with C. Tetanus anatoxin
antibiotic administration D. Oral polio vaccine (OPV)
C. Vaccination is contraindicated due to severe E. BCG vaccine
heart failure
D. Any vaccination is contraindicated due to
elderly age of the patient
E. Any vaccination is contraindicated due to
mitral valve disease
Хiрургiчний профiль 18

1. During medical examination a cadet 5. On the 4th day after recovery from a cold
in the naval college was detected to have a patient was hospitalized with complaints
a painless dense ulcer 1.5x0.5 in size in his of solitary spittings of mucoid sputum. On
perianal area at the 2 o’clock position. The the 2nd day there was a single discharge
ulcer floor resembles ”old fat”. What is the of about 250 ml of purulent blood-streaked
provisional diagnosis? sputum. Objectively: the patient’s condition
is moderately severe. Respiratory rate - 28-
A. Hard syphilitic chancre of the rectum 30/min., Ps- 96/min., BP- 110/70 mm Hg.
B. Rectal fissure Respiration over the left lung is vesicular,
C. Rectal fistula over the right lung - weakened. There are
D. Anal cancer various moist crackles over the lower lobe
E. Anal crypt suppuration and amphoric breath sounds near the angle
of scapula. What is the most likely diagnosis?
2. A 10-year-old boy, who was outdoors in
windy and cold weather, developed moderate A. Acute pulmonary abscess
pain and tingling in his fingers and toes. When B. Exudative pleuritis
he had returned home, his parents noticed that C. Acute focal pneumonia
the tips of his fingers and toes were white D. Pleural empyema
and their sensitivity was lost. The affected E. Pyopneumothorax
areas are warming up, the fingers are tingling
and in pain. Skin pallor changed into redness, 6. A 65-year-old woman on abdominal
tingling stopped, slight itching and swelling of palpation presents with a tumor in the
the fingers appeared. Determine the frostbite umbilical region and above it; the tumor is
degree in this child: 13x8 cm in size, moderately painful, non-
mobile, pulsing. On auscultation systolic
A. Frostbite of the I degree murmur can be observed. What is the most
B. Perniosis likely diagnosis?
C. Frostbite of the II degree
D. Frostbite of the III degree A. Abdominal aneurysm
E. Frostbite of the IV degree B. Gastric tumor
C. Arteriovenous aneurysm
3. 4 days after a patient received a gunshot D. Tricuspid insufficiency
wound to the soft tissues of middle third E. Bicuspid insufficiency
of the thigh, his condition suddenly began
deteriorating. There are complaints of bursting 7. After a case of purulent otitis a 1-year-old
pain in the wound; pain has been increasing boy has developed pain in the upper third of
during the last 12 hours. Edema of skin the left thigh, body temperature up to 39o C .
and hypodermic tissue quickly grows. Body Objectively: swelling of the thigh in its upper
temperature is 38,2o C , heart rate is 102/min. third and smoothed out inguinal fold. The limb
The wound edges gape, are dull in color; is in semiflexed position. Active and passive
the muscles, viable as of day before, now movements are impossible due to severe pain.
protrude into the wound, look boiled, are dull What diagnosis is the most likely?
in color, have dirty-gray coating, and fall apart
when held with forceps. What infection has A. Acute hematogenous osteomyelitis
developed in the wound? B. Acute coxitis
C. Intermuscular phlegmon
A. Anaerobic D. Osteosarcoma
B. Aerobic gram-negative E. Brodie’s abscess
C. Putrid
D. Aerobic gram-positive 8. A 74-year-old patient was delivered into
E. Diphtheria of the wound admission room with clinical presentations of
acute deep vein thrombosis of the shin. What
4. A patient received flame burns of both symptom is the most typical of this pathology?
hands. On the dorsal and palmar surface of
the hands there are blisters filled with serous A. Homans’ sign
fluid. The wrist joint region is hyperemic. B. Rovsing’s sign
The forearms were not injured. What is the C. Courvoisier’s sign
provisional diagnosis? D. Mayo-Robson’s sign
E. Grey Turner’s sign
A. II degree flame burn of the hands, 4% of
body surface area 9. A 50-year-old patient was delivered to a
B. II degree flame burn of the hands, 2% of hospital with complaints of blood traces in
body surface area urine. Urination is painless and undisturbed.
C. IIIa degree flame burn of the hands, 4% of Macrohematuria had been observed for 3
body surface area days. Objectively: kidneys cannot be palpated,
D. III degree flame burn of the hands, 4% of suprapubic area is without alterations,
body surface area external genitalia are non-pathologic. On
E. IIb degree flame burn of the hands, 2% of rectal investigation: prostate is not enlarged,
body surface area painless, has normal structure. Cystoscopy
revealed no alterations. What is the most likely
Хiрургiчний профiль 19

diagnosis? 14. A 58-year-old patient complains of pain


in the lower left extremity, which aggravates
A. Renal carcinoma during walking, and sensation of cold and
B. Bladder tuberculosis numbness in the both feet. The patient has
C. Varicocele been suffering from this condition for 6 years.
D. Dystopic kidney Objectively: the skin is pale and dry, with
E. Necrotic papillitis hyperkeratosis. On the left shin hair is scarce.
Pulse cannot be detected over the pedal and
10. A man complains of constant dull pain in popliteal arteries and is weakened over the
the perineum and suprapubic area, weak flow femoral artery. On the right limb pulsation of
of urine, frequent difficult painful urination, the popliteal artery is retained. What is the
nocturia. The patient has been suffering from most likely diagnosis?
this condition for several months, during which
urination was becoming increasingly difficult, A. Atherosclerosis obliterans of the lower
and pain in the perineum has developed. On extremities
rectal examination: the prostate is enlarged B. Obliterating endarteritis
(mainly its right lobe), dense, asymmetrical, C. Femoral artery thrombosis
central fissure is smoothed out, the right lobe D. Raynaud’s disease
is of stony density, painless, tuberous. What E. Buerger’s disease (thromboangiitis
disease is it? obliterans)
A. Prostate cancer 15. A 47-year-old woman came to the
B. Prostate sclerosis admission room with complaints of general
C. Urolithiasis, prostatolith of the right lobe weakness, dizziness, vomiting with blood
D. Prostate tuberculosis clots. Condition onset was 3 hours ago. The
E. Chronic congestion prostatitis patient has no preceding illnesses. Blood
pressure is 90/60 mm Hg, pulse is 106/min.,
11. A 17-year-old young man complains of of poor volume. The abdomen is soft, with
general weakness, trismus, twitching of the mild tenderness in the epigastrium. Blood
muscles in his left shin. 7 days ago he pierced
his foot with a nail. Objectively: at the sole test: erythrocytes - 2.1 · 1012 /L, Нb- 70 g/L,
of the foot there is a wound, 0,3х0,2 mm in hematocrit - 28%. What tactics should the
size, with small amount of serous-purulent doctor on duty choose?
discharge, the skin around the wound is A. Consult the surgeon
hyperemic. What is the most likely diagnosis? B. Refer the patient to the family doctor
A. Tetanus C. Give spasmolytics
B. Phlegmon D. Perform gastric lavage
C. Osteomyelitis E. Make an appointment for colonoscopy
D. Infected wound 16. A 52-year-old patient complains of pain
E. Erysipelas in the right part of her chest, dyspnea,
12. A patient with trauma of the lower third of cough with large amounts of foamy sputum
the forearm volar surface caused by a glass emitting foul smell and resembling ”meat
shard came to a first-aid center. Objectively: slops”. Objectively: the patient’s condition is
flexion of the IV and V fingers is impaired, grave, cyanosis is observed, breathing rate is
sensitivity of the inner dorsal and palmar 31/min., percussion sound above the right lung
surfaces of the hand and IV finger is decreased. is shortened, auscultation revealed various
What nerve is damaged? moist rales (crackles). What is the most likely
diagnosis?
A. Ulnar
B. Radial A. Lung gangrene
C. Median B. Lung abscess
D. Musculocutaneous C. Empyema of pleura
E. Axillary D. Multiple bronchiectasis
E. Chronic pneumonia
13. A man diagnosed with closed-angle
glaucoma, grade IIa, of the right eye is 17. A man complains of sore throat on the left,
registered for regular medical check-ups. In pain in his left ear, to up to 39o C , and nasal
the evening an acute glaucoma attack occurred sound of his voice. Disease onset was 5 days
in his right eye; an ambulance was called. What ago. Marked trismus and increased salivation
emergency aid would be optimal in this case? are observed. The head tilts to the left
shoulder. Left side of the soft palate presents
A. Pilocarpine, Diacarb (Acetazolamide), lytic with swelling, hyperemia, and infiltration.
mixture Retromandibular lymph nodes on the left are
B. Atropine eye drops acutely painful on palpation. Otoscopy results
C. Antibiotic eye drops, broad-spectrum are normal. Make the diagnosis:
D. Sulfacetamide sodium eye drops
E. Dexamethasone eye drops
Хiрургiчний профiль 20

A. Left-sided peritonsillar abscess A. Laparotomy


B. Retropharyngeal abscess B. Laparoscopy
C. Parapharyngeal phlegmon C. Cold to the abdomen
D. Peritonsillitis on the left D. Abdominal X-ray
E. Cervical phlegmon on the left E. Laparocentesis
18. A patient has the second and third degree 22. A 48-year-old woman was arrived to the
burns of the 15% of the body surface. On the surgical unit with wounds in her thigh. On
20th day after the trauma the patient presents examination the wound surface has dirty-
with sharp increase of body temperature, gray coating with unpleasant sweet smell.
general weakness, rapid vesicular respiration; Wound content resembles raspberry jelly. Skin
facial features are sharpened, BP is 90/50 mm tissues around the wound are glossy and turgid.
Hg, heart rate is 112/min. What complication Palpation reveals moderate crepitation in the
is it? tissues. What microflora is the most likely to
cause such inflammation?
A. Sepsis
B. Pneumonia A. Anaerobic clostridial
C. Acute intoxication B. Anaerobic non-clostridial
D. Purulent bronchitis C. Streptococci
E. Anaerobic infection D. Staphylococci
E. Blue pus bacillus
19. A patient in the state of clinical death is
being resuscitated through mouth-to-mouth 23. A 30-year-old patient was hospitalized
artificial pulmonary ventilation and external in an intensive care unit with a diagnosis of
cardiac massage. A doctor noticed that air does multiple bee stings. Skin is pale and covered
not flow into the patient’s airways and his head with cold sweat. Pulse can be palpated only at
and torso are positioned at the same level. Why the carotid arteries and is 110/min.; breathing
is artificial respiration ineffective in the given rate is 24/min., rhytmical, weakened. What
case? drug must be administered immediately?
A. Tongue retraction A. Epinephrine hydrochloride
B. Low breathing volume B. Prednisolone
C. External cardiac massage C. Norepinephrine hydrochloride
D. Probe is absent from the stomach D. Dopamine
E. The patient’s mouth is too small E. Tavegyl (Clemastine)
20. A patient complains of suppuration from 24. A 46-year-old woman has been hospitalized
the ear and impaired hearing of the left ear, with open fracture of the left thigh in its middle
which have been observed for the past 6 third. She underwent the surgery - fixation with
years. The patient had periodical headaches, extraosseous osteosynthesis plates. On the 4th
general indisposition, fever. Objectively: day after the surgery she developed pain in
otoscopy of the external auditory meatus the wound, body temperature rose over 39o C .
revealed mucopurulent odorless substance. What measures should be taken in this case?
The eardrum is of normal color, with central
perforation. What is the most likely diagnosis? A. Undo the sutures, drain the wound, and
prescribe antibiotics
A. Chronic mesotympanitis B. Prescribe broad spectrum antibiotics and
B. Otosclerosis hormonal agents
C. Acute otitis media C. Administer antibiotics intraosseously and
D. Chronic epitympanitis hypothermia locally
E. Chronic sensorineural hearing loss D. Inject antibiotics into the area surrounding
the wound, prescribe spasmolytics and
21. In 2 hours after a traffic accident a 28-year- analgesics
old man in grave condition was delivered to a E. Remove the fixation, prescribe
hospital. The patient complains of abdominal sulfanilamides
pain. He received a blow to the abdomen with
the steering wheel. Objective examination 25. A 42-year-old man was delivered to a
revealed the following: the abdomen does not surgical in-patient department with complaints
participate in respiration, is tense and acutely of icteric skin, pain in the right subcostal area.
painful on palpation; abdominal guarding is Biochemical blood analysis: total bilirubin -
present, peritoneal irritation signs are positive, 140 mcmol/l, direct bilirubin - 112 mcmol/l. On
hepatic dullness is absent. BP is 90/60 mm Hg, US: choledoch duct - 1,4 cm, a concrement is
heart rate is 120/min. What further treatment detected in the distal area. Gallbladder is 40
tactics should be chosen? cm, no concrements. What treatment tactics
should be chosen?
Хiрургiчний профiль 21

A. Endoscopic papillosphincterotomy A. Surgical intervention


B. Laparoscopic cholecystectomy B. Administration of spasmolytics
C. Laparotomy with choledoch duct drain C. Administration of ACE inhibitors
D. Laparotomy with cholecystectomy D. Kidney catheterization
E. Threatment in an infectious diseases hospital E. Administration of β -blockers
26. 4 weeks after myocardial infarction a 56- 30. A 49-year-old patient consulted a
year-old patient developed acute heart pain, doctor about difficult swallowing, voice
marked dyspnea. Objectively: the patient’s hoarseness, weight loss. These symptoms
condition is extremely grave, there is marked have been gradually progressing for the
cyanosis of face, swelling and throbbing of last 3 months. Objectively: the patient is
neck veins, peripheral pulse is absent, the exhausted, there are enlarged supraclavicular
carotid artery pulse is rhythmic, 130/min., BP is lymph nodes. Esophagoscopy revealed no
60/20 mm Hg. Auscultation of the heart reveals esophageal pathology. Which of the following
extremely muffled sounds, percussion reveals investigations is the most appropriate in this
heart border extension in both directions. case?
What is the optimal treatment tactics for this
patient? A. Computed tomography of chest and
mediastinum
A. Pericardiocentesis and immediate B. X-ray of lungs
thoracotomy C. Multiplanar imaging of esophagus
B. Oxygen inhalation D. Radioisotope investigation of chest
C. Puncture of the pleural cavity on the left E. Ultrasound investigation of mediastinum
D. Conservative treatment, infusion of
adrenomimetics 31. A 38-year-old patient has been delivered
E. Pleural cavity drainage by an ambulance to a surgical department with
complaints of general weakness, indisposition,
27. A 43-year-old woman complains of black stool. On examination the patient is pale,
persistent stomachache with recurrent pain there are dotted hemorrhages on the skin of his
attacks, nausea, repeated vomiting with torso and extremities. On digital investigation
stagnant bowel content, abdominal distension, there are black feces on the glove. Blood test:
and flatulence. She has been presenting with Hb- 108 g/L, thrombocytopenia. Anamnesis
these signs for 7 hours. Pulse is 116/min. states that similar condition was observed 1
The tongue is dry and brown. The abdomen year ago. Make the diagnosis:
is symmetrically distemded, soft, painful.
Percussion reveals tympanitis. On auscultation A. Thrombocytopenic purpura
there are bowel sounds with metallic overtone, B. Hemophilia
splashing, and dripping. Make the diagnosis: C. Bleeding from an ulcer
D. Rectal tumor
A. Acute intestinal obstruction E. Nonspecific ulcerative colitis
B. Acute necrotizing pancreatitis
C. Acute destructive cholecystitis 32. A 3-year-old child presents with sharp
D. Acute erosive gastritis deterioration of his general condition. He has
E. Acute nonspecific colitis a history of purulent otitis. His temperature
is now 38.5o C . The left leg is pressed to the
28. An infant cries during urination, the torso, active movements are absent, the lower
foreskin swells and urine is excreted in third of the thigh and knee joint are thickened,
drops. What approach to treatment should be hyperemic, with localized fever. Axial load
chosen? leads to acute discomfort of the patient. What
is the most likely diagnosis?
A. Create an opening into the foreskin cavity
B. Prescription of α-adrenergic blocking agents A. Epiphyseal osteomyelitis on the left
C. Prescription of antispasmodic agents B. Left hip fracture
D. Urinary bladder catheterization C. Rheumatoid arthritis
E. Epicystostomy D. Osteogenic sarcoma
E. Hygroma of the knee
29. A 20-year-old patient complains of pain in
the left lumbar region, arterial pressure rise 33. A patient after a blow to the head
up to 160/110 mm Hg. US revealed that the developed general symptoms of cerebral
structure and size of the right kidney were disturbance, nausea, vomiting, focal signs -
within age norms, there were signs of the 3rd hemi-hyperreflexia S>D, hemihyperesthesia
degree hydronephrotic transformation of the on the left, marked meningeal syndrome.
left kidney. Doppler examination revealed Neither cranial X-ray nor computer
an additional artery running to the lower tomography revealed any pathologies. What
pole of the kidney. Excretory urogram shows examination method would allow making and
a narrowing in the region of ureteropelvic clarification of the diagnosis?
junction. Specify the treatment tactics:
Хiрургiчний профiль 22

A. Lumbar puncture Cytoscopy detected hemiatrophy of the


B. Echoencephalography urinary bladder trigone, the right ureteral
C. Electroencephalography orifice is not detected. What pathology is it?
D. Angiography
E. Pneumoencephalography A. Agenesis of the right kidney
B. Dystopia of the right kidney
34. A burn victim with flame burns of the C. Hypoplasia of the right kidney
IIIA-B and IV degrees on his face, neck, and D. Agenesis of the right ureter
anterior surface of the thorax was brought E. Ectopic right ureteral orifice
into the admission room. The hairs in his
nostrils are burnt, his labial and glossal 38. A 64-year-old patient has been hospitalized
mucosa are gray-white. The voice is hoarse; with complaints of progressive jaundice
respirations are frequent and shallow; the that developed over 3 weeks without pain
patient has trumpet-like cough that produces syndrome and is accompanied by general
soot-streaked sputum. The signs of respiratory weakness and loss of appetite. Objectively:
failure were progressing, while the patient was temperature is 36,8o C , heart rate is 78/min,
being transported into the intensive care unit. abdomen is soft and painless, peritoneum
What emergency care must be provided to this irritation symptoms are not detected,
patient? palpation reveals sharply enlarged tense
gallbladder. What disease can be characterised
A. Intubation of the trachea and mechanical by these symptoms?
ventilation
B. Tracheostomy A. Cancer of pancreas head
C. Administration of bronchial spasmolytics B. Duodenal ulcer
D. Administration of respiratory analeptics C. Acute cholecystitis
E. Inhalation of moisturized oxygen D. Chronic cholecystitis
E. Cholecystitis caused by lambliasis
35. The body of a 24-year-old woman with
probable signs of poisoning has been found 39. A 37-year-old patient complains of pain
on the street. Forensic medical examination in the spinal column, reduced mobility. The
was requested by an investigator during condition persists for 7 years. ”Sway back” is
examination of the site and the body. observed, there is no movement in all spinal
According to the Criminal Procedure Code regions. On X-ray: ”bamboo spine” is detected.
currently in force in Ukraine, forensic medical What is the most likely diagnosis?
examination is required when it is necessary to
determine the: A. Ankylosing spondylitis
B. Osteochondrosis
A. Cause of death C. Spondylitis deformans
B. Manner of death D. Tuberculous spondylitis
C. Time of death E. Spondylolisthesis
D. Mode of death
E. Mechanism of death 40. A 54-year-old patient complains of
weakness, jaundice, itching skin. Disease onset
36. A 27-year-old man was hospitalized in was 1.5 months ago: fever up to 39o C appeared
severe condition 50 minutes after receiving a at first, with progressive jaundice developed 2
penetrating wound to the left side of the chest. weeks later. On hospitalisation jaundice was
Objectively the patient is in a stupor, his skin severely progressed. Liver cannot be palpated.
is pale and acrocyanotic. Pulse is 120/min., of Gallbladder is enlarged and painless. Blood
poor volume, weak. Blood pressure is 80/40 bilirubin is 190 mcmol/L (accounting mainly
mm Hg. Heart sounds are muffled, cardiac for direct bilirubin). Stool is acholic. What
borders are markedly expanded. In the III is the most likely reason for jaundice in this
intercostal area along the parasternal line on patient?
the left there is a stab-incised wound. Plain
chest X-ray shows enlarged heart shadow A. Mechanical jaundice
with smoothed out waist of the heart, there is B. Hepatocellular jaundice
hemothorax on the left to the 5th rib. What C. Hemolytic jaundice
contributes the most to the severity of the D. Caroli syndrome
patient’s condition? E. Gilbert’s syndrome
A. Cardiac tamponade 41. A 23-year-old man came to the surgeon
B. Acute heart failure with complaints of pain, redness of the skin,
C. Cardiac rhythm disturbance and swelling in the area of his proximal
D. Blood loss interphalangeal joint of the III finger on the
E. Hemothorax and acute respiratory failure right hand. Six days ago he pricked his finger
with a wire. Objectively the III finger on the
37. During regular examination of a 2-year- right hand is swollen, hyperemic, prominent in
old boy, he presents with enlarged left kidney, the projection of interphalangeal joint, sharply
painless on palpation. The right kidney painful on touch and during movements.
was undetectable on palpation. Excretory Finger mobility is reduced. Fluctuation sign
urography shows no contrast on the right.
Хiрургiчний профiль 23

is present. What diagnosis corresponds to the A. Esophageal tumor


given clinical presentation? B. Achalasia cardiae
C. Esophageal burns
A. Articular panaritium D. Diverticulum
B. Bone panaritium E. Barrett esophagus
C. Subcutaneous panaritium
D. Pandactylitis 46. A 45-year-old woman underwent one
E. Finger furuncle year ago mastectomy followed by chemo-
and radiation therapy. She now complains of
42. A 30-year-old patient was in a car accident. dyspnea at rest and temperature up to 37.2o C .
He is unconscious, pale, has thready pulse. In Her general condition is severe, acrocyanosis
the middle third of the right thigh there is is observed. The right side of her chest
an extensive laceration with ongoing profuse practically does not participate in respiration.
external arterial bleeding. What urgent actions Percussion reveals a dull sound below the
must be taken to save the life of the patient? 3rd rib; auscultation detects acute weakening
of the respiratory sounds. Pleural puncture
A. Tourniquet above the wound of the right on the right has yielded a large amount of
thigh hemorrhagic exudate. What complication has
B. Tourniquet below the wound of the right developed in the patient?
thigh
C. Artificial lung ventilation A. Carcinomatous pleuritis
D. Precordial thump B. Acute pleural empyema
E. Application of plaster bar C. Acute right-sided pleuropneumonia
D. Right lung abscess
43. A 25-year-old patient has been E. Pulmonary embolism
admitted to the hospital with the following
problems: weakness, sweating, itching, weight 47. A patient with signs of general
loss, enlarged submandibular, cervical, overexposure to cold presenting with local
axillary, inguinal lymph nodes. Objectively: frostbites of fingers has been delivered into an
hepatomegaly. Lymph node biopsy revealed admission room. Objectively: conscious, inert,
giant Berezovsky-Reed-Sternberg cells, speech is slow, the skin of the face is cold, body
polymorphocellular granuloma composed temperature is 34o C , heart rate is 68/min. What
of lymphocytes, reticular cells, neutrophils, would be the actions of a doctor on call?
eosinophils, fibrous tissue, and plasma cells.
What is the most likely diagnosis? A. Hospitalize the patient to the surgical
department
A. Lymphogranulomatosis (Hodgkin’s B. Hospitalize the patient to the therapeutics
lymphoma) department
B. Lymph node tuberculosis C. Hospitalize the patient to the traumatology
C. Lymphoreticulosarcoma department
D. Cancer metastases to the lymph nodes D. Let the patient go home
E. Macofollicular reticulosis E. Refer to a family doctor on the next day
44. A 68-year-old man complains of inability 48. A 24-year-old patient had been delivered
to urinate for a day. On attempt of urinary to a thoracic department with a chest injury,
bladder catheterization there was detected a fracture of the IV, V, VI ribs on the right.
rough stricture in the membranous portion of Plan radiography showed the fluid level in the
the urethra. What first aid tactics should be pleural cavity to be reaching the III rib on the
applied in this case? right. Puncture contained blood clots. What is
the optimal treatment tactics?
A. Troacar cystostomy
B. Adenomectomy A. Emergency thoracotomy
C. Optical internal urethrotomy B. Pleural puncture
D. α-adrenoblockers C. Thoracentesis and thoracostomy
E. Urinary bladder catheterization D. Hemostatic therapy
E. Medical thoracoscopy
45. Heart X-ray of a 31-year-old man has
revealed the following: with tightly filled 49. A 74-year-old patient visited a urologist
opacified esophagus there is a marginal filling with complaints of pain above the pubis and
defect in its middle third on the posterior wall; inability to urinate for 8 hours. At home he
the defect is 1,8x1,3 cm in size with clear oval had taken antispasmodics and had a warm bath
border. Mucosal folds are retained and envelop but no improvement occurred. Objectively:
the defect; wall peristalsis and elasticity are not abdomen is soft and painful above the pubis;
affected. There are no complaints regarding dullness of percussion sound is observed above
the condition of the patient’s alimentary canal. the pubis. Murphy’s (Pasternatski’s) punch
Make the provisional diagnosis: sign is negative on the both sides. What
condition does the patient have?
Хiрургiчний профiль 24

A. Acute urinary retention A. Sepsis


B. Paradoxal ischuria B. Purulent-resorptive fever
C. Chronic urinary retention C. Trombophlebitis
D. Anuria D. Meningitis
E. Oliguria E. Pneumonia
50. During preventive examination a 58-year- 54. A 28-year-old woman complains of girdle
old man on chest X-ray presents with multiple pain in her epigastric and left subcostal areas
globular pale shadows 3 cm in diameter within with irradiation to the back, nausea, and
parenchyma of the both lungs. Examination vomiting without relief. On examination a
in the oncologic hospital: the primary focus is surgeon observes stomach distension and
not found; transbronchial biopsy with cytologic meteorism. There are positive Mondor’s,
investigation detected cells of glandular Mayo-Robson’s, and Cullen’s symptoms. What
neoplasm. What tactics should the physician is the most likely diagnosis?
choose?
A. Acute pancreatitis
A. Polychemotherapy courses B. Acute cholecystitis
B. Exploratory laparotomy C. Acute intestinal obstruction
C. Exploratory thoracotomy D. Aortic dissecting aneurysm
D. Laparoscopy E. Splenic infarction
E. Symptomatic treatment at home
55. A man complains of high fever, pain in the
51. A 35-year-old man complains of persisting area of his right mastoid bone, and purulent
enlargement of his peripheral lymph nodes discharge from the right ear. One week ago
that cause him no discomfort. The case he had a case of URTI. Objectively the right
history states that the first lymph nodes auricle protrudes, the skin behind the ear is
to enlarge were cervical, supraclavicular, hyperemic and pastose; on palpation of the
and axillary; new groups of lymph nodes mastoid bone the pain intensifies; the auditory
emerged. Objectively the lymph nodes are soft meatus is filled with thick pus, posterosuperior
and elastic on palpation, enlarged, painless, meatal wall sags; the tympanic membrane is
not fixed to the surrounding tissue. What red and perforated. Make the diagnosis:
examination method would be the most
informative for early diagnostics of this A. Acute mastoiditis
disease? B. Furuncle of the external auditory meatus
C. Acute otitis media
A. Needle biopsy D. Acute otitis externa diffusa
B. Magnetic resonance tomography E. Exacerbation of chronic mesotympanitis
C. Radioisotope scanning of the skeleton
D. Ultrasound 56. A 28-year-old man after car accident
E. X-ray received a wound to the right side of his chest
inflicted by a sharp metal object. A foamy
52. A woman in her early- to mid-thirties has liquid flows out from the wound, there are
lost her consciousness 3-5 minutes ago. On tympanitis and acutely weakened respirations
examination: the skin is pale, no pulse over the in the right. Blood pressure is 70/30 mm Hg,
carotid arteries, no spontaneous respiration, pulse is 120/min., Hb is 28 g/L. X-ray shows
pupils are dilated; the patient is nonresponsive, collapsed right lung, horizontal fluid level is at
presents with atony. The patient’s condition the 3rd rib. What treatment tactics should be
can be determined as: chosen?
A. Apparent death A. Urgent thoracotomy
B. Natural death B. Delayed thoracotomy
C. Syncope C. Drain the right pleural cavity
D. Brain death D. Apply occlusive dressing to the wound
E. Coma E. Conservative therapy
53. A 38-year-old man underwent surgical 57. When her car collided with a tree, a 37-
treatment of a wound with a suppuration year-old woman felt sharp pain in her left hip
focus. On the 8th day after the procedure the joint. She was unable to get out of the car.
wound cleared of purulo-necrotic discharge Her position is forced, the hip is pressed to the
and granulations appeared. However, against abdomen, fixed, and rotated inwards; the limb
the background of antibacterial therapy, the is flexed in the knee, any attempt to change
patient’s body temperature persists as high the position results in sharp pain. Make the
as 38.5-39.5o C ; chills, excessive sweating, and diagnosis:
euphoria are observed in the patient; heart
rate is 120/min. What complication of the A. Closed dislocation of the left hip
local suppurative inflammatory process can be B. Contusion of the left hip joint
suspected? C. Hemarthrosis of the left hip joint
D. Arthritis of the left hip joint
E. Closed cervical fracture of the left hip
Хiрургiчний профiль 25

58. A 22-year-old man at 18:00 developed of treatment should be prescribed in this case?
persisting dull pain in the epigastrium. Three
hours later nausea appeared, he vomited A. Surgical
once. By the morning the pain shifted to the B. Conservative
right iliac area. Body temperature rose to C. Physical therapy
38.6o C , developed tachycardia of 110/min. D. Case monitoring
On examination there are muscle rigidity and E. Abstain from surgery in favor of complex
Bloomberg’s sign (rebound tenderness) in the conservative therapy
right iliac area of the anterior abdominal wall.
Plain x-ray of the abdomen shows no fluid 62. A 6-year-old girl arrived to the in-patient
levels, free air under the diapragm on the right. unit with complaints of enlargement of the
Make the diagnosis: lower third of her right thigh. According to the
case history, she has been stepping carefully
A. Perforation of a gastric ulcer on her right leg and limping for 6 months.
B. Renal colic Blood test detected anemia. X-ray of the right
C. Acute appendicitis thigh shows a round bone defect with clear
D. Acute pancreatitis margins resembling melting sugar in the distal
E. Acute cholecystitis metaphysis. What provisional diagnosis can be
made in this case?
59. On ultrasound of the thyroid gland, a 47-
year-old woman presents with a hypoechoic A. Osteogenic sarcoma of the right femur
node 1.6 cm in diameter with blurred margins B. Rheumatoid arthritis of the right knee joint
and intranodular hypervascularization. The C. Acute hematogenous osteomyelitis of the
doctor suspects thyroid carcinoma. What distal femoral metaphysis on the right
method should be used to verify the diagnosis? D. Tuberculous osteitis of the distal femoral
metaphysis on the right
A. Fine-needle aspiration biopsy E. Giant cell tumor of the right femur
B. Thyroid scintigraphy
C. Case monitoring 63. A 46-year-old man came to the surgeon’s
D. Determine TSH level in the blood office. He complains of twitching sensation
E. Positron emission tomography (PET) in the wound on his left foot, insomnia,
and anxiety. According to the patient, he
60. A 19-year-old young man complains of received this wound 5 days ago, when he
cough with expectoration of purulent sputum accidentally stepped on a glass shard, while on
in amount of 100 ml per day, hemoptysis, the beach. He requested no medical assistance.
dyspnea, increased body temperature up to Objectively the patient’s general condition is
37.8o C , general weakness, weight loss. The satisfactory, pulse is 75/min., blood pressure is
patient’s condition has been persisting for 4 130/80 mm Hg, temperature is 36.9o C . On the
years. Exacerbations occur 2-3 times per year. plantar surface of his foot there is a wound 1.5
The patient presents with malnutrition, pale cm long and up to 3 cm deep. The wound edges
skin, cyanosis of the lips, drumstick (clubbed) are moderately hyperemic, no discharge from
fingers. Tympanic percussion sound in the the wound is observed. What disease can be
lungs, weakened respiration, various numerous suspected in this patient?
moist crackles in the lower pulmonary
segments on the left can be observed. In A. Tetanus
B. Diphtheria
blood: erythrocytes - 3.2 · 1012 /L, leukocytes C. Anthrax
- 8.4 · 109 /L, ESR- 56 mm/hour. On X-ray: lung D. Fasciitis
fields are emphysematous, the left pulmonary E. Phlegmon
root is deformed and dilated. What is the most
likely diagnosis? 64. A 20-year-old student was brought to the
first-aid center. He has a closed fracture of the
A. Multiple bronchiectasis of the left lung left forearm and a contused lacerated wound
B. Chronic left-sided pneumonia on his left shin. After the patient received
C. Chronic abscess of the left lung initial wound management, he presented the
D. Left-sided pulmonary cystic dysplasia documents confirming that he has received
E. Suppuration of the cyst in the left lung all the necessary preventive vaccination as
scheduled. What should the doctor do to
61. An 8-year-old child presents with blood prevent tetanus in this patient?
pressure up to 180/100 mm Hg in the
upper limbs accompanied by headaches, A. Dynamic case monitoring
tinnitus, occasional nosebleeds, and high B. Administration of tetanus immunoglobulin
fatigability. On examination there is no C. Administration of anti-tetanus serum
pulse over the leg arteries. ECG shows left D. Antibiotic therapy
ventricular hypertrophy. MRI-scan shows E. Administration of tetanus toxoid
aortic narrowing to 5 mm in the typical place.
Coarctation of aorta is diagnosed. What kind
Педiатричний профiль 26

1. An 8-year-old boy developed a A. Surgical intervention


temperature of 37, 5o C two days after B. Continuation of conservative therapy
his recovery from the case of URTI. C. Physiotherapy
He complains of suffocation, heart pain. D. Sanatorium-and-spa treatment
Objectively: the skin is pale, tachycardia, the E. Physical training
I heart sound is weakened, short systolyc
murmur in the 4th intercostal area near 5. An infant has been born at the 41st week
the left edge of the breastbone. What of gestation. The pregnancy was complicated
heart disorder such clincal presentation is with severe gestosis of the second semester.
characteristic of? The weight of the baby is 2400 g, the height is
50 cm. Objectively: the skin is flabby, the layer
A. Nonrheumatic myocarditis of subcutaneous fat is thin, hypomyotonia,
B. Primary rheumatic carditis neonatal reflexes are weak. The internal
C. Myocardiodystrophy organs are without pathologic changes. This
D. Fallot’s tetrad newborn can be assessed as a:
E. Cardiomyopathy
A. Full-term infant with intrauterine growth
2. A 7-year-old boy has severe pulmonary retardation
mucoviscidosis (cystic fibrosis). He complains B. Premature infant
of dyspnea and blood expectoration. C. Immature infant
Objectively he presents with lagging physical D. Postmature infant
development, acrocyanosis, hepatomegaly, E. Full-term infant with normal body weight
drumstick fingers, and nail plates resembling
a ”clock face”. Provisional diagnosis of chronic 6. During examination a 4-month-old
pulmonary heart disease is made. What child with meningococcemia presents with
examination would be the most informative acrocyanosis, cold extremities, tachypnea, and
for diagnosis confirmation? thready pulse, blood pressure of 30/0 mm Hg,
anuria, and sopor. What clinical syndrome is
A. Doppler echocardiography it?
B. Electrocardiography
C. Chest X-ray A. Septic shock
D. Rheography of the pulmonary artery B. Neurotoxicosis
E. Ultrasound of the liver C. Exicosis
D. Encephalic syndrome
3. A 12-year-old boy with hypertrophic E. Acute renal failure
cardiomyopathy complains of dyspnea
caused by the slightest physical exertion. 7. The right arm of a newborn is stretched
Echocardiography detected asymmetrical along the torso with all its joints extended; the
left ventricular hypertrophy, signs of shoulder is rotated inwards, while the forearm
pulmonary hypertension, and left ventricular is pronated, the hand is in the position of
dilatation, its ejection fraction is 59%. These palmar flexion. Spontaneous movements are
developments are indicative of: absent in the shoulder and elbow joints,
passive movements are painless. What is the
A. Heart failure with preserved ejection most likely diagnosis?
fraction
B. Heart failure with reduced ejection fraction A. Duchenne-Erb palsy, superior proximal
C. Primary pulmonary hypertension type
D. Essential hypertension B. Dejerine-Klumpke palsy, inferior distal type
E. Symptomatic arterial hypertension C. Total obstetric palsy
D. Osteomyelitis of the right humerus
4. A 9-year-old boy has been suffering from E. Poliomyelitis
multiple bronchiectasis since he was 3 years
8. A 1-year-old child with a case of URTI
old. Exacerbations occur frequently (3-4 suddenly developed noisy respirations with
times a year), after conservative therapy difficult inspiration, intercostal retractions,
there are short remission periods. The and barking cough on the 2nd night after
disease progresses, the child is physically the disease onset. What is the most likely
underdeveloped, presents with pale skin, diagnosis?
acrocyanosis, deformed nail plates in the
shape of ”clock-face”. Bronchography reveals A. Stenosing laryngotracheobronchitis
saccular bronchiectases in the lower lobe of B. Acute pulmonary inflammation
the right lung. What further treatment tactics C. Bronchial asthma
should be chosen? D. Acute bronchitis
E. Acute bronchiolitis
9. A 10-year-old boy with symptoms of
arthritis and myocarditis was delivered into
a hospital. Based on clinical examination the
preliminary diagnosis of juvenile rheumatoid
Педiатричний профiль 27

arthritis was made. What symptom is the A. Mucoviscidosis (Cystic fibrosis)


most contributive for the diagnostics of this B. Acute obstructive bronchitis
disease? C. Recurrent obstructive bronchitis
D. Pertussis
A. Reduced mobility of the joints in the E. Acute obstructive pneumonia
morning
B. Regional hyperemia of the joints 13. A 10-year-old girl complains of
C. Affection of the large joints stomachache that appears and intensifies after
D. Enlarged heart she eats rough or spicy food, sour eructation,
E. Increased heart rate heartburn, frequent constipations, headaches,
irritability. She has been presenting with these
10. A 9-month-old child presents with fever, signs for 12 months. Her meals are irregular
cough, dyspnea. The symptoms appeared and consist of dry food. Objectively her diet is
5 days ago after a contact with a person sufficient in calories. The tongue is moist with
with URTI. Objectively: the child is in white coating near the root. The abdomen
grave condition. Temperature is 38o C , is soft and painful in the epigastrium. What
cyanosis of nasolabial triangle is present. method would be optimal for diagnosis-
RR- 54/min, nasal flaring during breathing making in this case?
is observed. There was percussion dullness
on the right below the scapula angle and A. Esophagogastroduodenoscopy
tympanic sound over the other areas of lungs. B. Intragastric pH-metry
Auscultation revealed bilateral fine moist C. Fractional gastric analysis (Fractional test
crackles predominating on the right. What is meals)
the most likely diagnosis? D. Phase-contrast X-ray imaging
E. Biochemical blood test
A. Pneumonia
B. URTI 14. A boy was born at 32 weeks of gestation. 2
C. Acute laryngotracheitis hours after the birth he developed respiratory
D. Acute bronchitis distress (RD). The RD severity assessed
E. Acute bronchiolitis by Silverman score was 5. The respiratory
disorders progressed, respiratory failure could
11. A 7-year-old boy has been an inpatient not be eliminated by Martin-Bouyer CPAP
for 1.5 months. He had been delivered to (continuous positive airway pressure). X-ray
the hospital with complaints of edemas of lungs shows reticular and nodular pattern,
all over his body, low urine output, and air bronchogram. What is the most likely
headache. Clinical urinalysis: proteins - cause of respiratory distress syndrome?
7.1 g/L, leukocytes - 1-2 in the vision
field, erythrocytes - 3-4 in the vision field. A. Hyaline membrane disease
During the course of treatment the edemas B. Segmental atelectasis
gradually dissipated, headache abated, C. Bronchopulmonary dysplasia
diuresis normalized. Daily urine proteins - D. Congenital pulmonary emphysema
3 g/L. Biochemical blood test: total protein - E. Edematous hemorrhagic syndrome
43.2 g/L, urea - 5.2 mmol/L, cholesterol - 9.2
mmol/L. What glomerulonephritis syndrome 15. An 8-year-old child with a 3-year-long
is the most likely to be present in the patient? history of diabetes was hospitalized in
hyperglycemic coma. Specify the initial dose
A. Nephrotic of insulin to be administered:
B. Nephritic
C. Isolated urinary A. 0,1-0,2 U/kg of body weight per hour
D. Hematuric B. 0,05 U/kg of body weight per hour
E. Mixed C. 0,2-0,3 U/kg of body weight per hour
D. 0,3-0,4 U/kg of body weight per hour
12. The mother of a 3-month-old child came E. 0,4-0,5 U/kg of body weight per hour
to a family doctor with complaints of her
child being physically underdeveloped and 16. A 5-year-old child that contacts with
suffering from cough attacks and dyspnea. viral hepatitis in the kindergarten presents
Anamnesis: the child is the result of the with increased body temperature up to
second full-term pregnancy with the risk of 38o C , weakness, low appetite, single case of
miscarriage (the first child died of pulmonary vomiting, dull pain in the subcostal area on
pathology at the age of 4 months, according the right. The child is provisionally diagnosed
to the mother). Body mass at birth is 2500 g. with viral hepatitis. What examination
Cough attacks were observed from the first would be the most informative for diagnosis
days of life, twice the child was treated for confirmation?
bronchitis. Considering the severity of the
child’s condition the doctor made the referral
for hospitalization. What diagnosis was most
likely stated in the referral?
Педiатричний профiль 28

A. ALT activity in blood A. Ciprofloxacin


B. Urine analysis for bile pigments B. Ampicillin
C. Feces analysis for stercobilin C. Cefazolinum
D. Blood test for bilirubin D. Azithromycin
E. Thymol turbidity test E. Chloramphenicol
17. An infant is 2.5 months old. The onset of 22. A 3-year-old girl is being treated at
the disease was gradual, the child had normal a resuscitation unit with diagnosis ”acute
body temperature but presented with slight kidney failure, oligoanuric stage”. ECG: high T
cough. Within a week the cough intensified, wave, extended QRS complex, displacement
especially at night; on the 12th day the child of S-T interval downwards below the isoline.
developed cough fits occurring up to 20 times What electrolyte imbalance is it?
per day and followed by vomiting. There was
one instance of respiratory arrest. Make the A. Hyperkalemia
diagnosis: B. Hypokalemia
C. Hypocalcemia
A. Pertussis D. Hypercalcemia
B. Parainfluenza E. Hyperphosphatemia
C. Congenital stridor
D. Respiratory syncytial infection 23. Mother of an 8-year-old girl complains
E. Adenovirus infection that the child is too short and has excessive
body weight. Objectively: obesity with fat
18. A 14-year-old boy presents with moderate deposits on the torso and face (round moon-
bronchial asthma in its exacerbation period. like face), acne, striae on the thighs and lower
What drug should be prescribed to stop an abdomen, hirsutism. What hormone can cause
acute attack of expiratory dyspnea? such symptoms, when in excess?
A. Salbutamol A. Cortisol
B. Cromolyn sodium (Cromoglicic acid) B. Thyroxin
C. Dexamethasone C. Testosterone
D. Lasolvan (Ambroxol) D. Insulin
E. Strophanthine (cardiac glycosides) E. Glucagon
19. An infant is 3 weeks old. Since birth there 24. A 9-year-old girl complains of fever up
has been observed periodical vomiting within to 38.5o C , headache, inertness, weakness,
a few minutes after feeding. The amount of loss of appetite, stomachache, and frequent
vomitive masses does not exceed the volume painful urination. Provisional diagnosis of
of previous feeding. The infant has age- acute pyelonephritis is made. Clinical urine
appropriate body weight. What is the most analysis: specific gravity - 1016, no protein,
likely cause of this symptom? leukocytes - 10-15 in the vision field. What
investigation method can verify the diagnosis
A. Pylorospasm of urinary tract infection?
B. Esophageal chalasia
C. Adrenogenital syndrome A. Bacteriological inoculation of urine
D. Pyloristenosis B. Rehberg test (creatinine clearance test)
E. Esophageal achalasia C. Zymnytsky test (density measurement of
20. A newborn with gestational age of 31 daily diuresis)
weeks presents with hypotonia and depressed D. Complete blood count
consciousness. Hematocrit is 35%, general E. Clinical urine analyses, dynamic testing
cerebrospinal fluid analysis shows increased 25. A newborn has Apgar score of 9. When
content of erythrocytes and protein, and should the infant be put to the breast?
low glucose. These data correspond with the
clinical presentation of: A. In the delivery room
B. After 12 hours
A. Intracranial hemorrhage C. After 2 hours
B. Meningitis D. On the 2nd day
C. Sepsis E. On the 3rd day
D. Anemia
E. Prenatal infection 26. A woman with blood group B(III)
21. From urine of a 14-year-old boy with Rh(+) gave birth to a full-term healthy boy.
the exacerbation of secondary obstructive Examination on the 3rd day of the infant’s life
pyelonephritis Pseudomonas aeruginosa was shows him to have icteric tint to his skin. The
isolated with a titer of 1000000 microbes per child has no problems with suckling, sleep
1 ml. What antibiotic is the most advisable in is nondisturbed. The abdomen is soft, the
this case? liver protrudes by 2 cm from under the costal
margin. Complete blood count: hemoglobin
- 200 g/L, erythrocytes - 5.5 · 1012 /L, total
Педiатричний профiль 29

bilirubin - 62 mcmol/L, indirect bilirubin - 52


mcmol/L. What condition can be suspected? A. Epidemic pemphigus of newborn
B. Syphilitic pemphigus
A. Physiologic jaundice C. Streptococcal impetigo
B. Congenital hepatitis D. Vulgar impetigo
C. Hemolytic disease of the newborn due to E. Atopic dermatitis
Rh incompatibility
D. Biliary atresia 31. A 1.5-month-old child on breastfeeding
E. Hemolytic disease of the newborn due to presents from birth with daily vomiting,
ABO incompatibility irregular liquid foamy feces, and flatulence,
which are resistant to antibacterial and
27. A 3-day-old infant with hyperbilirubinemia probiotic therapy; no increase of body mass
(428 mcmol/L) developed disturbances is observed. The child’s condition improved,
manifesting as periodical excitation and when breastmilk was substituted. What
convulsions against the background of pathology is it?
inertness, hypotension, hypodynamia,
and inhibition of unconditioned reflexes, A. Lactase deficiency
convergent strabismus, rotational nystagmus, B. Intestinal lambliasis (Giardiasis)
and setting-sun eye phenomenon. What is the C. Infectious enteritis
most likely cause of such symptoms? D. Drug-induced enteritis
E. Functional dyspepsia
A. Bilirubin encephalopathy
B. Craniocerebral injury 32. A 12-year-old girl after a case of
C. Brain tumor respiratory infection developed dyspnea at
D. Hydrocephalus rest, paleness of skin. Heart rate is 110/min.,
E. Infantile cerebral paralysis BP is 90/55 mm Hg. Heart sounds are muffled.
Borders of relative heart dullness: right - the
28. The left hand of a newborn is extended in parasternal line, upper - the III rib, left -
all its joints, stretched along the torso, and 1,0 cm outwards from the midclavicular line.
pronated in the forearm. Active movements Make the provisional diagnosis:
of the shoulder joint are retained. The hand
is flattened, atrophied, cold to touch, hangs A. Infectious myocarditis
passively. Grasping and Babkin’s reflexes B. Functional cardiopathy
are absent at the affected side. Hemogram C. Somatoform autonomic dysfunction
indicators are normal. Make the most likely D. Hypertrophic cardiomyopathy
diagnosis: E. Exudative pericarditis

A. Inferior distal obstetrical paralysis 33. A 5-year-old child has body temperature
B. Osteomyelitis risen up to febrile numbers, suffers from
C. Proximal obstetrical paralysis inertness, weakness. Examination revealed
D. Complete obstetrical paralysis hemorrhage on the skin of limbs and
E. Hypoxic-ischemic encephalopathy torso. Enlargement of cervical and axillary
lymph nodes can be detected. The liver
29. A 15-year-old girl complains of dizziness is 4 cm below the costal arch; the spleen
and sensation of lack of air that she develops is 6 cm below the costal arch. Blood test:
in emotionally straining situations. Relief erythrocytes - 2.3 · 1012 /L, Hb- 60 g/L,
occurs after she takes corvalol. Objectively: platelets - 40 · 109 /L, leukocytes - 32.8 · 109 /L,
hyperhidrosis and marble-like pattern of eosinophiles - 1%, band neutrophiles - 1%,
the skin of her palms and feet. Clinical segmented neutrophiles - 12%, lymphocytes
and instrumental examination revealed no - 46%, monocytes - 1%, blasts - 40%,
organic alterations of the central nervous, Duke’s bleeding time test result is 9 min.
cardiovascular, and respiratory systems. What What examination is necessary to make the
provisional diagnosis can be made? diagnosis?
A. Somatoform autonomic dysfunction A. Myelogram (bone marrow biopsy)
B. Obstructive bronchitis B. Lymph nodes biopsy
C. Bronchial asthma C. Abdominal US
D. Stenosing laryngotracheitis D. Detection of hepatitis markers
E. Acute epiglottitis E. Analysis of dynamic platelet function
30. 10 days after birth a newborn developed 34. ECG revealed the following in a 10-year-
sudden fever up to 38,1o C . Objectively: the old child: sharp acceleration of the heart rate
skin in the region of navel, abdomen and - 240/min., P wave overlaps with T wave
chest is erythematous; there are multiple and deforms it, moderate lengthening of PQ
pea-sized blisters with no infiltration at the interval, QRS complex is without alterations.
base; isolated bright red moist erosions with What pathology does this child have?
epidermal fragments are observed on the
periphery. What is the provisional diagnosis?
Педiатричний профiль 30

A. Paroxysmal atrial tachycardia A. Left-sided diaphragm paresis


B. Atrial hypertrophy B. Congenital pneumonia
C. Ventricular hypertrophy C. Left-sided pneumothorax
D. WPW syndrome D. Respiratory distress syndrome
E. Extrasystole E. Transient tachypnea of the newborn
35. A 13-year-old girl complains of fatigability, 39. A 12-year-old child had three attacks
frequent headaches, cardialgia. Eight years of acute rheumatic fever accompanied by
ago she had a case of pyelonephritis. Urine carditis. Examination revealed the symptoms
analyses periodically revealed leukocyturia. of chronic tonsillitis, mitral insufficiency, and
The child has undergone no further treatment. carious teeth. What is the optimal method of
On examination: increased BP up to 150/100 secondary prophylaxis?
mm Hg. Ultrasound investigation revealed
significant reduction of the right kidney. What A. Year-round bicillin prophylaxis until the age
process is leading in arterial hypertension of 25
pathogenesis in this case? B. Course of cardiotrophic drugs twice a year
C. Year-round bicillin prophylaxis for 3 years
A. Hyperactivity of renin-angiotensin system D. Tonsillectomy
B. Disturbance of water-electrolytic balance E. Oral cavity sanitation
C. Disturbance of renal circulation
D. Hypersympathicotonia 40. A 3-year-old child has been delivered to a
E. Increased cortisol level hospital with complaints of pain in the legs,
fever, loss of appetite. Objectively: pale skin
36. An 9-year-old child was hospitalized and mucosa, hemorrhagic rash. Lymph nodes
for fever up to 39,8o C , inertness, moderate are enlarged, painless, dense and elastic, not
headache, vomiting. Examination revealed matted together. Bones, joints, and abdomen
meningeal symptoms. Lumbar puncture are painful. The liver and spleen are enlarged.
was performed. The obtained fluid was Hemogram: Hb- 88 g/l, color index - 1,3,
characterised by increased opening pressure, platelets - 80 · 109 /l, leukocytes - 25, 8 · 109 /l,
was transparent, with the cell count of 450 lymphoblasts - 70%, ESR- 52 mm/hour. Make
cells per 1 mcL (mainly lymphocytes - 90%), the provisional diagnosis:
glucose level of 3,6 mmol/l. What agent could
have caused the disease in the child? A. Acute leukemia
B. Thrombocytopenic purpura
A. Enterovirus C. Acute rheumatic fever
B. Neisseria meningitidis D. Infectious mononucleosis
C. Mycobacterium tuberculosis E. Hemorrhagic vasculitis (Henoch-Schonlein
D. Staphylococcus aureus purpura)
E. Streptococcus pneumoniae
41. During last several weeks an 11-year-
37. A 22-day-old infant developed old girl has been complaining of dyspnea
subcutaneous red nodes from 1.0 to 1.5 cm in and edema of shins and feet after physical
size on the scalp; later the nodes suppurated. exercise. After a long rest or sleep through the
Temperature increased up to 37.7o C , night her edemas diminish significantly. On
intoxication symptoms appeared, regional clinical examination there are enlarged liver
lymph nodes enlarged. Complete blood count: and rasping systolic murmur over the cardiac
anemia, leukocytosis, neutrocytosis, increased area. Blood and urine analyses are without
ESR. What diagnosis will you make? changes. What is the most likely cause of the
child’s edema?
A. Pseudofurunculosis
B. Pemphigus A. Heart failure
C. Vesiculopustulosis B. Angioneurotic edema
D. Scalp phlegmon C. Acute pyelonephritis
E. - D. Hepatocirrhosis
E. Nephrotic syndrome
38. A full-term newborn (born with the body
weight of 3900 g at gestational age of 39 42. An 8-year-old girl with complaints
weeks) on the first day of his life developed of painful urination, frequent low-volume
respiratory disturbances: dyspnea, arrhythmic urination, and leukocyturia was diagnosed
respiration, cyanosis attacks. On examination with acute cystitis. 10 days before the
there is paradoxical respiration observed and disease onset she was treated by the
left side of the chest lags behind in the act of gynecologist for acute vulvitis. 5 days ago
breathing. On auscultation the respiration is she presented with mild catarrhal symptoms.
weakened in the lungs on the left. Neurologist Her mother ascribes the child’s disease to her
diagnosed the patient with left-sided Erb- overexposure to cold. Specify the most likely
Duchenne palsy. Complete blood count shows infection route:
no changes. What is the most likely diagnosis?
Педiатричний профiль 31

A. Ascending between doses of vaccine in this case?


B. Descending
C. Hematogenic A. 1 month
D. Contact B. 6 months
E. Lymphogenic C. 2 months
D. 3 months
43. A 5-year-old boy complains of severe E. 12 months
dyspnea and sensation of lack of air.
Objectively the child assumes orthopneic 46. A 6-month-old infant is not vaccinated.
position, presents with marked peripheral The physician recommends a DPT
cyanosis, drumstick fingers, nail plates (diphtheria, pertussis, tetanus) vaccination
resembling a ”clock face”, the borders of but the mother is absolutely against this
cardiac dullness are bilaterally extended; procedure. Choose the most substantial
coarse systolic murmur can be detected over argument in favor of vaccination:
the whole surface of the heart and is especially
pronounced in the II intercostal area on the A. Risk of lethal consequences
left near the sternum. What disease can be B. Epidemic risk for the others
characterized by such presentations? C. Personal professional experience
D. High quality of vaccines
A. Fallot’s tetrad E. -
B. Dilated cardiomyopathy
C. Defect of the interventricular septum 47. During administration of planned DPT
D. Primary bacterial endocarditis vaccination the child suddenly developed
E. Defect of the interatrial septum acute anxiety, signs of pain response, dyspnea,
grunting respirations, cutis marmorata, cold
44. A 10-year-old boy is delivered into a sweat. Objectively the child’s consciousness
polytrauma unit after he received a blunt is disturbed, heart rate is 150/min., blood
trauma of the thorax, having fallen from pressure is 60/40 mm Hg, heart sounds
the bicycle. Upon hospitalization his blood are muffled. The child was diagnosed with
pressure is 110/80 mm Hg, heart rate is anaphylactic shock. What drug should be
96/min. Chest X-ray is noncontributive to the administered first?
diagnosis. Echocardiogram shows free liquid
in the pericardial cavity, in the amount of up A. Epinephrine
to 100 ml. In an hour after the hospitalization B. Lasix (Furosemide)
the patient started to develop increasing signs C. Suprastin (Chloropyramine)
of heart failure: jugular venous distention, D. Euphylline (Aminophylline)
decreased blood pressure down to 90/70 E. Analgin (Metamizole)
mm Hg, tachycardia up to 120/min. On
auscultation muffled heart sounds. What 48. Among first-year schoolchildren there was
would be the primary tactics of a physician? a case of measles registered. A 7-year-old
boy from the same group was not vaccinated
A. Pericardiocentesis (pericardial puncture) against measles due to refusal of his parents.
B. Cardiac glycosides intravenously His clinical history has no cases of measles
C. Constant oxygenotherapy in the past and is not contraindicatory to
D. Diuretics intravenously immunobiological agents. Choose the most
E. Antibiotics intravenously rational tactics of measles prevention in this
schoolboy:
45. A healthy child 1 year and 5 months of
age is being vaccinated against hepatitis B. A. Measles-Mumps-Rubella vaccine
The child did not receive the first dose of B. Isolation for 20 days
the vaccine previously, while in the maternity C. Antiviral agents
hospital. The doctor makes an individual D. Antibiotics
vaccination schedule for this child and plans E. Immunomodulators
the administration of the next dose of
the vaccine. What is the minimum interval
Акушерство i гiнекологiя 32

1. A 28-year-old woman has been weight of the parturient woman is 62


delivered to a hospital with acute pain in the kg. Fetus is in a longitudinal lie, the
lower abdomen. There was a brief syncope. head is engaged to the pelvic inlet. Belly
The delay of menstruation is 2 months. circumference is 100 cm. Uterine fundus
Objectively: the patient has pale skin, BP- height is 35 cm. What body weight of the
90/50 mm Hg, Ps- 110/min. Lower abdomen fetus can be expected?
is extremely painful. Vaginal examination
reveals uterus enlargement. Promtov’s A. 3.5 kg
sign (pain during bimanual gynecological B. 4 kg
examination) is positive. Right uterine C. 2.5 kg
appendages are enlarged and very painful. D. 3 kg
Posterior vault hangs over. What is the most E. 4.5 kg
likely diagnosis? 6. A 22-year-old woman, gravida 1, para 0
A. Right-sided tubal pregnancy arrived with complaints of sharply painful
B. Right ovary apoplexy contractions that occur every 4-5 minutes
C. Acute right-sided salpingoophoritis and last for 25-30 seconds. Amniotic fluid did
D. Pelvioperitonitis not burst. The fetus is in transverse lie, fetal
E. Incipient abortion heartbeats are not affected. Contraction ring
is acutely painful, located obliquely at the
2. A parturient woman complains of pain in umbilicus. What is the most likely diagnosis?
her mammary gland. In the painful area there
is an infiltration 3x4 cm in size with softened A. Impending uterine rupture
center. Body temperature is 38,5o C . What is B. Uterine tetany
the most likely diagnosis? C. Excessive uterine activity during labor
D. Discoordinated labor
A. Acute suppurative mastitis E. Uterine rupture
B. Pneumonia
C. Pleurisy 7. A 30-year-old multigravida has been
D. Milk retention in labour for 18 hours. 2 hours ago the
E. Birth trauma pushing stage began. Fetal heart rate is clear,
rhythmic, 136/min. Vaginal examination
3. A 29-year-old woman came to a reveals complete cervical dilatation, the fetal
gynecologist with complaints of irritability, head in the pelvic outlet plane. Sagittal
tearfulness, headache, nausea, occasional suture is in line with obstetric conjugate,
vomiting, pain in the heart area, tachycardia the occipital fontanel is near the pubis. The
attacks, memory impairment, meteorism. patient has been diagnosed with primary
These signs appear 6 days before uterine inertia. What is the further tactics
menstruation and disappear the day before of labour management?
menstruation or during its first 2 days. On
vaginal examination: the uterus and uterine A. Outlet forceps
appendages are without alterations. What B. Labour stimulation
diagnosis is the most likely? C. Cesarean section
D. Skin-head Ivanov’s forceps
A. Premenstrual syndrome E. Vacuum extraction of the fetus
B. Algodismenorrhea
C. Ovarian apoplexy 8. A 20-year-old woman on the 10th day
D. Genital endometriosis after her discharge from the maternity ward
E. Neurosis developed fever up to 39o C and pain in her
left mammary gland. On examination the
4. A primigravida at the term of 20 weeks mammary gland is enlarged, in its upper
complains of pain in her lower abdomen, outer quadrant there is a hyperemic area. In
smearing blood-streaked discharge from the this area a dense spot with blurred margins
genital tracts. Uterine tone is increased, can be palpated. The patient presents with
fetus is mobile. On vaginal examination: the lactostasis and no fluctuation. Lymph nodes
uterus is enlarged according to the term, in the right axillary crease are enlarged and
uterine cervix is shortened to 0,5 cm, external painful. Specify the correct diagnosis:
cervical orifice is open by 2 cm. What is the
most likely diagnosis? A. Lactational mastitis
B. Abscess
A. Risk of late abortion with hemorrhage C. Erysipelas
B. Risk of late abortion without hemorrhage D. Dermatitis
C. The process of late abortion E. Tumor
D. Incomplete late abortion
E. Attempted late abortion 9. A 16-year-old girl has primary
amenorrhea, no pubic hair growth, normally
5. The pregnancy is full term. The body developed mammary glands; her genotype is
Акушерство i гiнекологiя 33

46 ХY; uterus and vagina are absent. What is appendages can be detected. What is the
your diagnosis? most likely diagnosis?
A. Testicular feminization syndrome A. Sheehan’s syndrome (postpartum
B. Mayer-Rokitansky-Kuster-Hauser hypopituitarism)
syndrome B. Ovarian amenorrhea
C. Cushing syndrome C. Turner’s syndrome
D. Sheehan syndrome D. Ovarian exhaustion syndrome
E. Cushing disease E. Galactorrhea-amenorrhea syndrome
10. 6 hours ago the waters of a 30-year- 14. A 28-year-old woman complains of
old gravida 1, para 0, burst; her preliminary increased intervals between menstruations,
period was pathologic and lasted for over 2 up to 2 months, and hirsutism. Gynecological
days; the term of pregnancy is 39 weeks. No examination revealed the following: ovaries
labor activity is observed. Fetal head presents are enlarged, painless, and dense; no
above the pelvic inlet. Fetal heartbeats are alterations of the uterus. US of the lesser
142/min., clear and rhytmic. On vaginal pelvis: ovaries are 4-5 cm in diameter, with
examination the uterine cervix is not dilated. numerous enlarged follicles on the periphery.
What further tactics should the doctor X-ray of the skull base: sellar region is
choose? widened. What is the most likely diagnosis?
A. Perform cesarean section A. Stein-Leventhal syndrome (polycystic
B. Induce cervical dilation with prostaglandins ovarian syndrome)
C. Stimulate the labor with oxytocin B. Algodismenorrhea
D. Wait for the onset of spontaneous labor C. Sheehan syndrome (postpartum
E. Prolong the pregnancy, while providing hypopituitarism)
antibacterial treatment D. Premenstrual syndrome
11. A 23-year-old woman came the the E. Morgagni-Stewart-Morel syndrome
gynecologist with complaints of blood smears (metabolic craniopathy)
from her genital tracts that have been
observed for a long time. Her menstruation 15. The gynecology unit received a patient
has been delayed for 8 weeks. Examination with uterine bleeding that started 6 hours
shows the uterine body to be enlarged up after induced abortion at the term of 11-12
to 14 weeks of pregnancy. US detected weeks. Objectively the skin is pale, pulse
a vesicular mole. What tactics should the is 100/min., blood pressure is 100/70 mm
doctor choose? Hg. On vaginal examination the uterus is
painless, its enlargement corresponds to the
A. Curettage of the uterine cavity 10th week of pregnancy; uterine cervix is
B. Hormonal treatment dilated enough to let in one finger, there
C. Hemostatic treatment are fragments of the fertilized ovum. What
D. Supravaginal uterine amputation actions should be taken next:
E. Uterectomy
A. Urgent repeated curettage of the uterine
12. A 46-year-old woman came to a maternity cavity
clinic with complaints of moderate blood B. Uterotonic drugs
discharge from the vagina, which developed C. Treatment for acute anemia
after the menstruation delay of 1,5 months. D. Antibacterial agents
On vaginal examination: the cervix is clean; E. Prescribe rest and continue to monitor the
the uterus is not enlarged, mobile, painless; patient’s condition
appendages without changes. Make the
diagnosis: 16. A woman came to the general
practitioner with complaints of fatigability,
A. Dysfunctional uterine bleeding significant weight loss, weakness, and loss
B. Adenomyosis of appetite. She has been presenting with
C. Ectopic pregnancy amenorrhea for the last 8 month. One year
D. Submucous uterine myoma ago she gave birth to a live full-term child.
E. Cancer of the uterine body Blood loss during delivery was 2 liters.
The woman received blood transfusion and
13. A 30-year-old woman complains of blood components. What is the most likely
amenorrhea that lasts for 2 years after she diagnosis?
has given birth, loss of hair and body weight.
The labor was complicated with hemorrhage
caused by uterine hypotonia. Objectively
the patient is of asthenic type, her external
genitalia are hypoplastic, the uterine body
is small in size and painless. No uterine
Акушерство i гiнекологiя 34

A. Sheehan’s syndrome (postpartum A. Juvenile uterine hemorrhage


hypopituitarism) (Dysfunctional)
B. Stein-Leventhal syndrome (polycystic B. Hypomenstrual syndrome
ovary) C. Inflammation of uterine appendages
C. Turner’s syndrome (Pelvic inflammatory disease)
D. Homologous blood syndrome D. Pelviperitonitis
E. Somatoform autonomic dysfunction E. Endometritis
17. A 22-year-old woman complains of 21. A 15-year-old adolescent girl came
itching and profuse discharge from her the the gynecologist with complaints of
genital tracts. The condition developed painful menstruations that are accompanied
10 days ago after a sexual contact. by nausea, vomiting, and dizziness. Her
Bacterioscopy of a discharge sample menarche was at 12. Menstruations became
detected trichomonads. What drug should painful since she was 14, remain regular.
be prescribed for treatment in this case? What treatment should be prescribed in this
case?
A. Metronidazole
B. Ampicillin A. Analgesics, antispasmodics,
C. Erythromycin antiprostaglandine therapy
D. Zovirax (Acyclovir) B. Antiinflammatory treatment only
E. Valcyclovir C. Antihemorrhagic agents
D. Antiandrogen therapy
18. A multigravida on the 38th week of her E. Vitamin supplements
pregnancy complains of increased BP up to
140/90 mm Hg, edema of the shins for 2 22. A 20-year-old woman, gravida 2, para 1
weeks. In the last month she gained 3.5 kg has been in labor for 4 hours. Her condition is
of weight. Urine analysis: protein - 0.033 g/L. satisfactory. Moderately painful contractions
Make the diagnosis: occur every 3 minutes and last for 35-40
seconds. The waters have not burst yet.
A. Mild preeclampsia The fetus is in longitudinal position. Fetal
B. Moderate preeclampsia heartbeats are 136/min., clear and rhytmic.
C. Pregnancy hypertension Major segment of the fetal head is engaged to
D. Severe preeclampsia the pelvic inlet. Vaginal examination shows
E. Pregnancy edema smooth cervix of 6cm, amniotic sac is intact,
sagittal suture is in the left oblique diameter,
19. A 37-year-old woman complains of acute occipital fontanel is on the right near the
pain in the genital area, swelling of the symphysis pubis. What stage of the labor is
labia, pain when walking. Objectively: body it?
temperature is 38,7o C , Ps- 98/min. In the
interior of the right labia there is a dense, A. Active phase of the first stage of normal
painful tumor-like formation 5,0x4,5 cm in labor
size, the skin and mucous membrane of B. Latent phase of the first stage of normal
genitals are hyperemic, there is profuse foul- labor
smelling discharge. What is the most likely C. The second stage of normal labor
diagnosis? D. Precursors of childbirth
E. Preliminary stage
A. Acute bartholinitis
B. Labial furuncle 23. A multigravida at 39 weeks of gestation
C. Acute vulvovaginitis presenting with regular labour activity for 8
D. Bartholin gland cyst hours has been delivered to a hospital; the
E. Carcinoma of vulva waters broke an hour ago. She complains
of headache, seeing spots. BP is 180/100
20. A 14-year-old girl has been delivered to mm Hg. Urine test results: protein - 3,3
a gynecological department with complaints g/l, hyaline cylinders. Fetal heart rate is
of profuse blood discharge from her genital 140/min, rhythmical. Vaginal examination
tract for 2 weeks. Anamnesis: menstruation reveals complete cervical dilatation, the fetal
since 13, irregular, painful, profuse; the last head is on the pelvic floor, sagittal suture is in
one was 2 months ago. Objectively: pale line with obstetric conjugate, the occipital
skin and mucosa, BP- 100/60 mm Hg, Hb- fontanel is under the pubis. What is the
108 g/l. The abdomen is soft and painless optimal tactics of labour management?
on palpation. Rectal examination revealed
no pathologies of reproductive organs. What A. Outlet forceps
condition is it? B. Cavity forceps
C. Cesarean section
D. Vacuum extraction of the fetus
E. Conservative labour management
Акушерство i гiнекологiя 35

24. An 18-year-old woman complains of pain A. Pituitary tumour


in her lower abdomen, profuse purulent B. Lactational amenorrhea
discharge from the vagina, temperature C. Stein-Leventhal syndrome (polycystic
rise up to 37,8o C . Anamnesis states that ovary syndrome)
she had a random sexual contact the D. Sheehan’s syndrome (postpartum
day before the signs appeared. She was hypopituitarism)
diagnosed with acute bilateral adnexitis. E. Cushing’s disease
On additional examination: leukocytes are
present throughout all vision field, bacteria, 28. A 30-year-old woman complains of
diplococci with intracellular and extracellular milk discharge from her breasts and no
position. What is the most likely agent in the menstruation for the last 5 months. One
given case? physiologic childbirth was 4 years ago.
There are no maldevelopments of mammary
A. Neisseria gonorrhoeae glands. Bimanual examination revealed
B. Escherichia coli diminished uterus and normal sized ovaries.
C. Chlamydia trachomatis MRI-scan shows no brain pathologies.
D. Trichomona vaginalis Thyroid-stimulating hormone is within
E. Staphylococcus aureus normal limits. Serum prolactin is high. What
is the most likely diagnosis?
25. It is the 3rd day after the normal term
labor; the infant is rooming-in with the A. Hyperprolactinemia
mother and is on breastfeeding. Objectively: B. Hypothyroidism
the mother’s general condition is satisfactory. C. Polycystic ovaries
Temperature is 36.4o C , heart rate is 80/min., D. Pituitary adenoma
BP is 120/80 mm Hg. Mammary glands E. Sheehan’s syndrome (postpartum
are soft and painless; lactation is moderate, hypopituitarism)
unrestricted milk flow. The uterus is dense,
the uterine fundus is located by 3 fingers 29. A 25-year-old woman during self-
width below the navel. Lochia are sanguino- examination detected a tumor in the upper
serous, moderate in volume. Assess the external quadrant of her right mammary
dynamics of uterine involution: gland. On palpation: painless, dense, mobile
growth 2 cm in diameter is detected in
A. Physiological involution the mammary gland; no changes in the
B. Subinvolution peripheral lymph nodes are observed. On
C. Lochiometra US of the mammary glands: in the upper
D. Pathologic involution external quadrant of the right mammary
E. Hematometra gland there is a space-occupying lesion of
increased echogenicity 21х18 mm in size. The
26. A 6-year-old girl came to a general most likely diagnosis is:
practitioner with her mother. The child
complains of burning pain and itching in A. Fibrous adenoma
her external genitalia. The girl was taking B. Breast cyst
antibiotics the day before due to her suffering C. Diffuse mastopathy
from acute bronchitis. On examination: D. Breast cancer
external genitalia are swollen, hyperemic, E. Mastitis
there is white deposit accumulated in the
folds. The most likely diagnosis is: 30. A 45-year-old woman came to the
maternity clinic with complaints of periodical
A. Candidal vulvovaginitis pains in her mammary glands that start
B. Trichomoniasis 1 day before menstruation and stop after
C. Nonspecific vulvitis the menstruation begins. Palpation of the
D. Helminthic invasion mammary glands detects diffuse nodes
E. Herpes vulvitis predominantly in the upper outer quadrants.
What is the most likely diagnosis?
27. A 22-year-old woman complains of
amenorrhea for 8 months. Anamnesis A. Fibrocystic mastopathy
states that menarche occured at the age of B. Breast cancer
12,5. Since the age of 18 the patient has C. Mastitis
a history of irregular menstruation. The D. Hyperprolactinemia
patient is nulligravida. The mammary glands E. Breast cyst
are developed properly, nipples discharge
drops of milk when pressed. Hormone test: 31. A woman complains of temperature
prolactin level is 2 times higher than normal. increase up to 39o C , sharp pains in her lower
CT reveals a bulky formation with diameter abdomen, and sanguinopurulent discharge
of 4 mm in the region of sella. What is the from her genital tracts. From her case history
most likely diagnosis? it is known that 6 days ago she underwent
illegal abortion. Objectively her blood
Акушерство i гiнекологiя 36

pressure is 100/60 mm Hg, pulse is 110/min. A. Early delivery


Abdominal rigidity, rebound tenderness B. Delivery at 37 weeks of gestation
(Bloomberg’s sign), and painful palpation C. Screening for Rh-antibodies in 2 weeks
of the lower abdomen are observed. On and urgent delivery in case of further increase
bimanual examination the uterus is enlarged of antibody titer
up to 7 weeks of pregnancy, painful, and soft; D. Introduction of anti-Rh (D)
posterior vaginal fornix overhangs. Make the immunoglobulin
diagnosis: E. Ultrasound for signs of hemolytic disease
of the fetus
A. Pelviperitonitis
B. Endometritis 35. A 55-year-old woman came to a
C. Acute adnexitis gynecologist with complaints of leukorrhea
D. Pyosalpinx and bloody discharge from the vagina after
E. Metroendometritis 5 years of menopause. Anamnesis states
no pregnancies. Bimanual examination: the
32. A 14-year-old girl came to a general uterus and uterine appendages are without
practitioner with complaints of weakness, changes. During diagnostic curettage of the
loss of appetite, headache, rapid fatigability. uterine cavity the physician scraped off
Her last menstruation was profuse and enchephaloid matter. What is the most likely
lasted for 14 days after previous delay of 2 diagnosis in this case?
months. Objectively: the skin is pale, heart
rate is 90/min., BP is 110/70 mm Hg, Hb A. Endometrial carcinoma
is 88 g/l. Rectal examination: the uterus B. Adenomyosis
and its appendages are without changes, C. Subserous uterine myoma
no discharge from the genital tracts. What D. Cervical carcinoma
complication occurred in the patient? E. Ovarian carcinoma
A. Posthemorrhagic anemia 36. A 24-year-old pregnant woman on her
B. Somatoform autonomic dysfunction of 37th week of pregnancy has been delivered to
hypotonic type a maternity obstetric service with complaints
C. Migraine of weak fetal movements. Fetal heartbeats
D. Gastritis are 95/min. On vaginal examination the
E. Dysmenorrhea uterine cervix is tilted backwards, 2 cm
long, external orifice allows inserting a
33. A postpartum woman on the 12th day fingertip. Biophysical profile of the fetus
after the normal delivery complains of pain equals 4 points. What tactics of pregnancy
localized in her left gastrocnemius muscle. management should be chosen?
Body temperature is 37,2o C ; pulse is 85/min,
rhythmic; blood pressure is 128/80 mm Hg. A. Urgent delivery via cesarean section
Mammary glands are soft and painless. The B. Treatment of placental dysfunction and
uterus is behind the pubis. The left leg in repeated analysis of the fetal biophysical
the area of gastrocnemius muscle is by 3 profile on the next day
cm larger than the right leg in the diameter. C. Doppler measurement of blood velocity in
Internal organs present no pathologies. What the umbilical artery
complication can be suspected? D. Urgent preparation of the uterine cervix
for delivery
A. Deep vein thrombosis of the shin E. Treatment of fetal distress, if ineffective,
B. Iliofemoral thrombosis then elective cesarean section on the next day
C. Varicose veins of lower extremities
D. Endometritis 37. During regular preventive gynecological
E. Myositis examination a 30-year-old woman was
detected to have dark blue punctulated
34. Examination of a Rh-negative pregnant ”perforations” on the vaginal portion of
woman at 32 weeks of gestation revealed a the uterine cervix. The doctor suspects
four-time rise of Rh-antibody titer within endometriosis of the vaginal portion of the
2 last weeks; the titer is 1:64. The first two uterine cervix. What investigation method
pregnancies resulted in antenatal fetal death would be most informative for diagnosis
due to hemolytic disease. What is the optimal confirmation?
tactics of pregnancy management?
A. Colposcopy, target biopsy of the cervix
B. US of the lesser pelvis
C. Hysteroscopy
D. Curettage of the uterine cavity
E. Hormone testing
38. A 26-year-old woman came to a
gynecologist for a regular check-up. She has
Акушерство i гiнекологiя 37

no complaints. Per vaginum: the uterus lies second pregnancy. She complains of fever,
in anteflexion, not enlarged, dense, mobile, chills, nausea, vomiting, lumbar pain, and
painless. On the left from the uterus in dysuria. Costovertebral angle tenderness is
the area of uterine appendages there is present on both sides. Urine analysis: pyuria,
a mobile painless outgrowth that can be bacteriuria. Blood test: leukocytosis. What is
moved independently from the uterus. On the most likely diagnosis?
the right the appendages cannot be detected.
What additional investigation would be A. Gestational pyelonephritis
informative for diagnosis clarification? B. Cystitis
C. Pyelitis
A. Ultrasound of the lesser pelvis D. Glomerulonephritis
B. Metrosalpingography E. Latent bacteriuria
C. Examination for urogenital infection
D. Colposcopy 42. A 32-year-old pregnant woman at the
E. Colonoscopy term of 5-6 weeks was vaccinated against
influenza along with her whole family. At
39. A postparturient woman, who has that time she was not aware of her pregnancy.
been breastfeeding for 3 weeks, made The pregnancy is wanted. The woman needs
an appointment with the doctor. For the an advice from the family doctor regarding
last 6 days she has been feeling unwell, the maintenance of her pregnancy, namely
complains of body temperature of 38- whether there is a risk of fetal malformations
39o C , general weakness; within the last because of received vaccination. What advice
2 days she developed pain and redness should the doctor give in this case?
in the area of her right mammary gland.
Examination revealed her mammary gland A. Vaccination against influenza is safe during
to be significantly enlarged and deformed; pregnancy
breast tissue fluctuations and lymphadenitis B. Therapeutic abortion is recommended
are observed. What type of mastitis is the C. Immediate ultrasound of the lesser pelvis
most likely? is necessary
D. Test for antibodies against influenza virus
A. Phlegmonous mastitis is necessary
B. Serous mastitis E. An infectious diseases specialist must be
C. Infiltrative mastitis consulted
D. Lactostasis
E. Mammary edema 43. A 17-year-old girl has made an
appointment with the doctor. She plans to
40. On the 9th day after childbirth the begin her sex life. No signs of gynecological
obstetric patient developed high fever up pathology were detected. In the family
to 38o C . She complains of pain in the right history there was a case of cervical cancer
mammary gland. The examination revealed that occurred to the patient’s grandmother.
the following: a sharply painful infiltrate can The patient was consulted about the
be palpated in the right mammary gland, the maintenance of her reproductive health.
skin over the infiltrate is red, subareolar area What recommendation will be the most
and nipple are swollen and painful. What is helpful for prevention of invasive cervical
your diagnosis? cancer?
A. Abscess of the right mammary gland A. Vaccination against human papillomavirus
B. Mastopathy (HPV)
C. Cancer of the right mammary gland B. Vitamins, calcium, omega-3
D. Serous mastitis C. Immunomodulators
E. Fibrous cystic degeneration of the right D. Antiviral and antibacterial drugs
mammary gland E. Timely treatment of sexually transmitted
diseases
41. A woman is on the 32nd week of her
Гiгiєна та органiзацiя охорони здоров’я 38

1. A regional cardiologist is given a task The complex of symptoms indicates the


to develop a plan for preventive measures initial stage of chronic intoxication with:
aimed at decreasing cardiovascular
mortality rates. What measures should be A. Lead
planned for secondary prevention? B. Manganese
C. Mercury
A. Prevention of recurrences and D. Tin
complications E. Ethanol
B. Referring patients for sanatorium-and-spa
treatment 6. A 10-year-old girl exhibits high level
C. Prevention of diseases of physical development ( + 3σ ), her
D. Referring patients for inpatient treatment body length increased by 10 cm within a
E. Optimization of lifestyle and living year (which is double the norm for her
conditions age group), the number of permanent
teeth corresponds with the age norm (20),
2. Caries morbidity rate is 89% among the development of her secondary sex
residents of a community. It is determined characteristics is three years ahead of her
that fluorine content in water is 0.1 mg/L. age (Ма, Р, Ах, Menarche). Development
What preventive measures should be taken? rate ahead of her biological age can occur
A. Water fluorination due to:
B. Tooth brushing A. Endocrine disorders
C. Fluorine inhalations B. Acceleration
D. Sealant application C. Certain components of her diet
E. Introduce more vegetables to the diet D. Sports training
3. A 39-year-old man, a battery attendant, E. Deficient hygienic education
suddenly developed weakness, loss of 7. Examination of a group of persons
appetite, nonlocalized colicky abdominal living on the same territory revealed
pains, and nausea. Objectively his skin the following common symptoms: dark-
is gray; there is pink-gray stripe on his yellow pigmentation of the tooth enamel,
gums; the stomach is soft and sharply diffuse osteoporosis of bone apparatus,
painful. Blood test detected erythrocytes ossification of ligaments and joints,
with basophilic stippling and anemia. functional disorders of the central nervous
The patient has a history of peptic ulcer system. This condition may be caused by
disease of the stomach. There is tendency the excessive concentration of the following
to constipation. What is the most likely microelement in food or drinking water:
provisional diagnosis?
A. Fluorine
A. Saturnism (lead poisosning) B. Copper
B. Acute appendicitis C. Nickel
C. Perforation of gastric ulcer D. Iodine
D. Acute cholecystitis E. Cesium
E. Chronic alcoholism
8. A worker of a blowing shop complains
4. A 9-month-old infant presents with of headache, irritability, sight impairment
delayed tooth eruption and fontanel closure, - he sees everything as if through a ”net”.
weakness, and excessive sweating. What Objectively: hyperemic sclera, thickened
type of hypovitaminosis is the most likely cornea, decreased opacity of pupils, visual
in this child? acuity is 0.8 in the left eye, 0.7 in the
A. Hypovitaminosis D right eye. The worker uses no means of
B. Hypovitaminosis C personal protection. What diagnosis is the
C. Hypovitaminosis B1 most likely?
D. Hypovitaminosis B6 A. Cataract
E. Hypovitaminosis A B. Conjunctivitis
5. During the periodic medical examination C. Keratitis
an assembly fitter (works on soldering D. Blepharospasm
E. Progressive myopia
details) didn’t report any health problems.
Closer examination revealed signs of 9. The inpatient surgery unit has introduced
asthenic-vegetative syndrome. Blood the method of laparoscopic cholecystectomy
included red blood cells with basophilic into its practice. As the result the average
aggregations and a somewhat higher duration of postoperative care provided
number of reticulocytes, urine had a high to the patients could be reduced to
concentration of delta-aminolevulinic acid. 3.4±0.8 days compared to 7.3±1.1 days
Гiгiєна та органiзацiя охорони здоров’я 39

that were required after non-laparoscopic A. Lead and lead salts


cholecystectomy. What method of medical B. Tin
statistics can confirm the statistical C. Carbon monoxide
significance of the difference between these D. Nitric oxide
two estimates? E. Zinc
A. Calculation of Student’s confidence 13. Due to introduction of a new treatment
interval method, average duration of therapy in
B. Calculation of correlation coefficient the experimental group was 12.3±0.2 days
C. Calculation of standardized ratio compared to 15.4±0.4 days in the control
D. Calculation of average values (measures group that was treated by the old method.
of central tendency) What calculations should be made to
E. Calculation of relative values estimate the statistical significance of the
difference in the results?
10. In April during the medical examination
of various population groups, 27% of A. T-test (Student’s t-distribution)
individuals presented with low working B. Sign test (Z-test)
ability and rapid fatigability. The following C. Matching factor (chi-squared test)
symptoms were observed in the affected D. Wilcoxon T-test
individuals: swollen friable gingiva that E. Kolmogorov-Smirnov test
bleeds when pressed, hyperkeratosis
follicularis not accompanied by skin dryness. 14. A patient, who had eaten canned
These symptoms most likely result from the mushrooms (honey agaric) three days ago,
following pathology: developed vision impairment (diplopia,
mydriasis), speech disorder, disturbed
A. C -hypovitaminosis swallowing. What type of food poisoning
B. Parodontosis occurred in the patient?
C. A-hypovitaminosis

D. 1 -hypovitaminosis A. Botulism
E. Polyhypovitaminosis B. Food toxicoinfection
C. Fusariotoxicosis
11. During regular check-up the doctor D. Honey agaric poisonong
examines a young woman, a student, with E. Lead salts poisoning
height of 162 cm and weight of 59 kg. She
complains that in the evening she becomes 15. Establishments participating in
unable to see clearly the objects around her. medical examinations include: medical and
Objectively her skin is dry, presents with preventive treatment facilities, hygiene and
hyperkeratosis. Her daily ration has the preventive treatment facilities, sociomedical
following vitamin content: vitamin A - 0.5 expert committees, Ministry of Defence
mg, vitamin B1 - 2.0 mg, vitamin B2 - 2.5 mg, medical committees, Ministry of Domestic
vitamin B6 - 2 mg, vitamin C - 70 mg. Make Affairs medical committees, forensic
the diagnosis: medicine agency, etc. Specify what service
deals with sociomedical assessment of
A. A-hypovitaminosis temporary disability:
B. B1 -hypovitaminosis
C. B2 -hypovitaminosis A. Medical and preventive treatment
D. B6 -hypovitaminosis facilities
E. C -hypovitaminosis B. Hygiene and preventive treatment
facilities
12. A man works in casting of nonferrous C. Sociomedical expert committees
metals and alloys for 12 years. In the D. Ministry of Defence medical committees
air of working area there was registered E. Ministry of Domestic Affairs medical
high content of heavy metals, carbon committees
monoxide, and nitrogen. During periodic
health examination the patient presents with 16. During assessment of work conditions
asthenovegetative syndrome, sharp pains at the factory manufacturing mercury
in the stomach, constipations, pain in the thermometers, the content of mercury
hepatic area. In urine: aminolevulinic acid vapors in the air of the working area
and coproporphyrin are detected. In blood: is revealed to exceed the maximum
reticulocytosis, low hemoglobin level. Such concentration limit. Specify the main
intoxication is caused by: pathway of human body exposure to
mercury:
Гiгiєна та органiзацiя охорони здоров’я 40

A. Respiratory organs test results: Hb- 90 g/l, anisocytosis,


B. Intact skin poikilocytosis. The most likely causative
C. Damaged skin factor of this condition is the inadequate
D. Gastrointestinal tract intake of:
E. Mucous tunics
A. Iron
17. During health assessment of car drivers B. Copper
and police officers on point duty, the C. Zinc
physicians detected carboxyhemoglobin in D. Magnesium
the blood of the patients, weakened reflex E. Selene
responses, disturbed activity of a number
of enzymes. Revealed professional health 22. To assess the effectiveness of medical
disorders are most likely to be associated technologies and determine the power and
with the effect of: direction of their effect on the public health
indicators, the research was conducted to
A. Carbon monoxide study the immunization rate of children and
B. Sulfurous anhydride measles incidence rate by district. What
C. Mental stress method of statistical analysis should be
D. Aromatic hydrocarbons applied in this case?
E. Nitric oxide
A. Calculation of correlation coefficient
18. In the process of hiring, a prospective B. Calculation of morbidity index among the
employee has undergone preventive nonvaccinated
medical examination and was declared C. Calculation of coefficient of agreement
fit to work in this manufacturing D. Calculation of standardized ratio
environment. What type of preventive E. Calculation of statistical significance of
medical examination was it? the difference between two estimates
A. Preliminary 23. Posture of an 11-year-old boy was
B. Scheduled determined during preventive examination.
C. Periodical The child presents with curled forward
D. Specific rounded shoulders, the head is bowed
E. Comprehensive forward, the thorax is flattened, the stomach
is bulging. In the vertebral column there
19. An employee has been sick for 4 months, are deepened cervical and lumbar flexures.
further treatment is necessary, the patient What posture does the child have?
is unable to work. Who is authorized to
provide further disability examination of A. Kyphosis
this patient? B. Lordosis
C. Stooping
A. Sociomedical expert committee D. Corrected
B. Medical consultative board E. Normal
C. Physician in charge and the head of the
department 24. The objective of a statistical research was
D. Chief physician of a medical facility to find out to what extent the population
E. Deputy chief responsible for disability peruses the available medical services. For
examination this purpose 300 residents of the area were
interviewed. Information was collected by
20. On laboratory investigation of a pork means of a special questionnaire. What
sample there is 1 dead trichinella detected method of collecting information was used
in 24 sections. This meat should be: by the researchers?
A. Handed over for technical disposal A. Anamnestic (history-taking)
B. Allowed for sale with no restrictions B. Immediate registration
C. Processed and sold through public catering C. Immediate examination
network D. Doing extracts
D. Processed for boiled sausage production E. -
E. Frozen until the temperature of - 10o C is
reached in the deep layers, with subsequent 25. A 47-year-old man is employed at
exposure to cold for 15 days the weaving workshop, has 15-year-long
record of service at this factory; his
21. Examination of a 43-year-old man work conditions are associated with high-
objectively revealed pallor of skin and frequency and high-intensity noise. During
mucous membranes, loss of tongue papillae, periodical examination he was diagnosed
transverse striation of fingernails, cracks with occupational deafness. What are the
in the mouth corners, tachycardia. Blood grounds for making such a diagnosis?
Гiгiєна та органiзацiя охорони здоров’я 41

A. Audiometry data and hygienic assessment A. Primary healthcare


of working environment B. Emergency aid
B. Record of service at this factory C. Secondary healthcare
C. Noise characteristic at this factory D. Tertiary healthcare
D. Central nervous system examination E. Palliative care
results
E. Inner ear examination results 30. 40-50 minutes after the completion
of repair works conducted in a closed
26. Estimation of community health level garage, with car engine running, the repair
involved analysis of a report on diseases workers developed severe headache in the
registered among the population of district temporal area, nausea, tinnitus, vertigo, etc.
under charge (reporting form 12). What These symptoms are characteristic of acute
index is calculated based on this report? poisoning with:
A. Prevalence A. Carbon monoxide
B. Index of pathological affection B. Aldehydes
C. Index of morbidity with temporary C. Organochlorides
disability D. Hydrogen sulfide
D. Index of hospitalized morbidity E. Fluoride
E. Index of basic non-epidemic morbidity
31. A 45-year-old veterinary worker has
27. A 60-year-old man has a diet consisting made an appointment with the doctor
of unvaried food staples: mostly cereals, for regular examination. In his duties he
potato, pasta; few vegetables and little fats frequently deals with animals, however
(especially animal fats). During medical he denies working with rabies-affected
examination he complains of deterioration animals. Previously he has received no
of his twilight vision. This condition can be antirabic vaccination. What should the
caused by lack of: doctor recommend in this case?
A. Retinol A. Preventive immunization with antirabic
B. Amino acids vaccine
C. Fats B. Vaccination in case of contact with sick
D. Calcium animal
E. Carbohydrates C. Preventive immunization with rabies
immunoglobulin
28. A 52 year old man came to see his D. Administration of antirabic vaccine and
family physician complaining of pain in the rabies immunoglobulin
chest. After taking history and performing E. Preventive immunization with anti-rabies
physical exam the doctor decided to direct serum
the patient to cardiologist for a consultation.
What level of medical care is being proposed 32. A 28-year-old woman has made
to the patient? an appointment with the family doctor
to receive vaccination against influenza.
A. Secondary healthcare However, having collected the patient’s
B. Emergency healthcare medical history, the doctor claimed this
C. Primary healthcare procedure to be absolutely contraindicated
D. Tertiare healtcare for this woman. What anamnestic data is the
E. Palliative care absolute contraindication to vaccination?
29. A 30-year-old woman made an A. Egg white intolerance
appointment with the family doctor for B. Pregnancy at 30 weeks
scheduled vaccination of her 2-year-old C. Blood hemoglobin - 109 g/L
child. What type of healthcare provides such D. Body temperature - 37.2o C
medical services? E. Psoriasis in the remission phase

You might also like